assh hand 1998

118
1. Mobilization programs after flexor tendon repair not only promote gliding of the tendons and decrease adhesion formation but also: 1) have no effect on tendon healing. 2) increase loss of water and glycosaminoglycans. 3) increase collagen fibril size and improve tendon strength. 4) increase the likelihood of repair rupture. 5) lose repair strength over the first three weeks and must be carefully monitored. Preferred Response: 3 Discussion: It has been shown that gliding and stress of flexor tendons after repair not only keeps the tendon from losing strength postoperatively, as they do when kept immobile, but the healing milieu is markedly improved. Collagen fibril size increases and improves tendon strength. Water and glycosaminoglycan content increases and increases tendon strength. There is no decrease in tendon strength over the first three weeks atter repair, so there is no increase in tendon rupture. 2. A 28-year-old athletic woman with juvenile rheumatoid arthritis presents with a sudden loss of ring finger MCP extension, wrist pain, and the X-rays shown in Figure 1. Recommended treatment is: 1) Darrach procedure 2) extensor tendon grafting 3) tenodesis of the EDC-ring finger to the adjacent EDC 4) Suave-Kapandji procedure and EDC tenodesis 5) synovectomy DRUJ and extensor tendon reconstruction

Upload: mona-ahmed

Post on 10-Aug-2015

59 views

Category:

Documents


1 download

TRANSCRIPT

Page 1: ASSH Hand 1998

1. Mobilization programs after flexor tendon repair notonly promote gliding of the tendons and decreaseadhesion formation but also:

1) have no effect on tendon healing. 2) increase loss of water and glycosaminoglycans. 3) increase collagen fibril size and improve tendonstrength. 4) increase the likelihood of repair rupture. 5) lose repair strength over the first three weeks andmust be carefully monitored.

Preferred Response: 3

Discussion:

It has been shown that gliding and stress of flexortendons after repair not only keeps the tendon fromlosing strength postoperatively, as they do when keptimmobile, but the healing milieu is markedly improved.Collagen fibril size increases and improves tendonstrength. Water and glycosaminoglycan contentincreases and increases tendon strength. There is nodecrease in tendon strength over the first three weeksatter repair, so there is no increase in tendon rupture.

2. A 28-year-old athletic woman with juvenile rheumatoidarthritis presents with a sudden loss of ring finger MCPextension, wrist pain, and the X-rays shown in Figure 1.Recommended treatment is:

1) Darrach procedure 2) extensor tendon grafting 3) tenodesis of the EDC-ring finger to the adjacent EDC 4) Suave-Kapandji procedure and EDC tenodesis 5) synovectomy DRUJ and extensor tendon reconstruction

Page 2: ASSH Hand 1998

Preferred Response: 4

Discussion:

This patient with juvenile rheumatoid arthritis has developed acaput ulna syndrome with rupture of the extensor digitorumcommunis to the ring finger. Treatment should includetenodesis of the ruptured EDC to the adjacent extensortendon. Extensor tendon grafting is less reliable and isreserved for cases in which three or more tendons areruptured. Additionally, the patient's distal radioulnar jointsubluxation should be treated. Synovectomy and retinacularreconstruction is usually unsuccessful because of the poorquality of the tissues which results in resubluxation. TheDarrach procedure is the traditional treatment of choice, butcan exacerbate ulnar subluxation of the carpus, which isalready evident in this young patient. Also, rupture of theextensor tendons is a reported complication of the Darrachprocedure. The Sauve-Kapandji operation is thought to offeradditional support to the carpus and thus help to prevent thiscomplication.

3. Following laceration and primary repair of the posteriorinterosseous nerve at the level of the proximal supinator theexpected order of return of function is:

1) SUP, ECU, EDC, EDQ, EPL, EIP 2) SUP, EDU, EDQ, ECU, EPL, EIP 3) SUP, ECU, EDQ, EDC, EIP, EPL 4) SUP, EDC, ECU, EDQ, EPL, EIP 5) SUP, EDC, ECU, EDQ, EIP, EPL

Page 3: ASSH Hand 1998

Preferred Response: 4

Discussion:

Understanding radial nerve motor branch anatomy is necessaryfor performing surgery in its vicinity, undertaking nerve repair,forecasting location of a compression lesion, and predicting rateand order of recovery. Radial nerve motor innervation order andits variability is controversial. Sunderland pointed out thataccurate prediction of reinnervation order and rate depends notonly on measurement of the motor branch points along the mainnerve but also the additional distances along the branches tomuscle entry points. Other studies that do not take this intoaccount show different results. In the most recent study, Abramset. al. utilizing 20 cadaver specimens found anatomic patterns thatcorrespond to those of Sunderland, providing the most accurateand common order of innervation as illustrated in Figure 2. 4. Of the following muscles, which one is least affected byVolkmann's contracture?

1) Pronator teres. 2) Flexor digitorum profundus. 3) Flexor pollicis longus. 4) Flexor carpi ulnaris. 5) Flexor digitorum superficialis

Preferred Response: 4

Discussion:

In Volkmann's contracture the most severely affected muscles are

Page 4: ASSH Hand 1998

the flexor digitorum profundus and flexor pollicis longus,followed by the flexor digitorum superficialis and pronator teres.There is much less injury to the wrist flexors and extensors andbrachioradialis.

5. A 45-year-old female sustained a puncture wound to her palmfrom a cactus plant. Five days later, she developed an ulceration atthe puncture site and Iymphangitis with nodules along the volarforearrn. Two weeks later, these nodules ulcerate. Culturesshould be taken and antibiotic treatment should include:

1) Dicloxacillin 500 mg po q six hours 2) Cefazolin I gm IV q eight hours 3) Acyclovir 400 mg po q eight hours 4) Potassium Iodide .3 mg po tid 5) Amphotericin B 50 mg IV q day

Preferred Response: 4

Discussion:

Fungi are often overlooked as a source of hand infections.Sporotrichosis is the most common subcutaneous fungalinfection found in North America and predominately affects theupper extremities. Implantation can be caused by a variety ofplants in addition to the frequently recognized rose thorn,including cactus and sphagnum moss. The infection is difficult todiagnose on the basis of standard stains and definitive diagnosisrequires cultures. Treatment of choice is a saturated solution ofpotassium iodide (Igm/ml) taken .3 mg po tid after meals andincreased as tolerated to 1.5 gm po tid.

6. The Jamar dynamometer five-rung test:

1) Permits subjective assessment of the motivation and sincerityof effort of the patient. 2) Is not a reliable test because it is dependent on patientmotivation. 3) Cannot be used to accurately assess patients suspected of lowsustained grip effort. 4) Shows flatter curves for sincere effort. 5) Rarely results in bell-shaped curves in strong patients.

Preferred Response: 1

Discussion:

Page 5: ASSH Hand 1998

A number of methods to detect patients who purposely exert loweffort during group evaluations have been devised. The Jamardynamometer has been shown using a five-rung test todemonstrate objective answers to the subjective questions ofmotivation and effort. A bellshaped curve is correlated withappropriate effort and a flatter curve with feigning a response.

7. A 46-year-old male factory worker who performs martial artsas an avocation presents with a four-month history of wrist pain.Examination reveals dorsal wrist synovitis and tenderness.Radiographs are shown in Figures 3A and 3B. The most likelydiagnosis is:

1) Kienbock's disease 2) Preiser's disease 3) Accessory ossicle of lunate 4) Volar fracture dislocation of lunate 5) Lunate fracture

Preferred Response: 5

Discussion:

Although rare, fresh lunate fractures, as distinguished fromKienbock's disease, do occur. Teisen and Hjarbaek divided theseinjuries into five groups based on the location of the fracture.Because important nutrient vessels and stabilizing ligamentsattach to the palmar pole of the lunate, displaced fractures of thepalmar pole and transverse fractures through the mid portionwould appear to be best treated by open reduction and internalfixation. Minimally displaced fractures of the dorsal pole, sagittalfractures, and small marginal fractures should be treatednon-surgically.

Page 6: ASSH Hand 1998

8. Two years after traumatic brain injury, a 37-year-old male witha severe spastic flexion contracture of the hand and wristundergoes STP transfer, wrist flexor tenotomy, and wristarthrodesis. Postoperatively he has persistent MP flexioncontractures. What is the most likely cause of this deformity?

1) Inadequate lengthening with the STP transfer. 2) Failure to release contracted MP joint capsules. 3) Unopposed lumbrical pull. 4) Failure to release the intrinsics. 5) Failure to excise the proximal carpal row when performing thearthrodesis.

Preferred Response: 4

Discussion:

Severe spastic flexion contractures of the hand and wrist maybenefit from STP transfer, FPL lengthening and wrist arthrodesiswith or without proximal row carpectomy. Intrinsic tightness maybe responsible for MP contractures. Intrinsic release orneurectomy of the motor branch of the ulnar nerve will helpprevent or correct this deformity.

9. Intrinsic spasticity of the fingers and thumb after closed headinjury results in an inability to:

1) Adduct the thumb. 2) Extend the interphalangeal joint of the thumb. 3) Flex the metacarpophalangeal joint of the fingers. 4) Extend the metacarpophalangeal joints of the fingers. 5) Extend the proximal interphalangeal joints of the fingers.

Preferred Response: 4

Discussion:

Intrinsic tightness of the thumb results in a position of metacarpaladduction (from tightness of the adductor pollicis, first dorsalinterosseous and thenar musculature), MCP joint flexion (fromtightness of the thenar muscles) and narrowing of the first webspace. Extrinsic FPL tightness produces IP flexion deformity ofthe thumb. Intrinsic tightness of the fingers results in a positionof fixed flexion of the MCP, and may lead to the development of

Page 7: ASSH Hand 1998

swan-neck deformity at the PIP and DIP joints.

10. A 45-year-old farmer "jams" his non-dominant ring finger.He complains of pain and modest swelling about the PIP jointthat gradually resolves. Examination six weeks later reveals a 30°extensor lag at the PIP joint. Active flexion is normal and passivemotion is 0° to 95°. Recommended treatment is:

1) PIP joint extension splint for six weeks. 2) Central slip repair. 3) Buddy taping of ring finger to long finger. 4) Arthrodesis of PIP joint. 5) Lateral band release.

Preferred Response: 1

Discussion:

The patient sustained a closed injury to the central slip when hejammed his finger. Splinting the PIP joint in full extension,allowing active and passive DIP motion, is likely to allow healingof the central tendon rupture with appropriate gliding of thelateral bands. Only if this first line conservative treatment failsshould operative intervention be considered. Primary shorteningof the central tendon can be considered as could division of theextensor mechanism over the middle phalanx. Lateral bandrelease in the face of full passive range of motion is not needed toobtain motion and will remove a portion of the extensormechanism needed for PIP and DIP motion. Buddy taping isrecommended for PIP collateral ligament injuries but is nothelpful for extensor injuries

11. A 56-year-old patient with a 42-week history of upperextremity pain is suspected of having reflex sympatheticdystrophy. The most accurate way to make a diagnosis of RSDis:

1) clinical history and physical examination 2) three-phase bone scan 3) intravenous phentolamine test 4) thermography 5) isolated cold-stress testing

Preferred Response: 1

Page 8: ASSH Hand 1998

Discussion:

The most accurate way to make a diagnosis of RSD is on thebasis of the clinical history and physical exam. A 1984 report byMackinnon and Holder implied that a negative three-phase bonescan essentially ruled out RSD. Subsequent reports havechallenged this concept. Many patients with clinical RSDparticularly for more than 26 weeks may have a negativethree-phase bone scan. Thermography, isolated cold-stresstesting, and intravenous phentolamine are all considerednon-specific tests.

12. The artery of origin of a dorsoradial pedicled bone graft forthe management of scaphoid nonunions is:

1) posterior interosseous 2) dorsal intercarpal arch 3) posterior division of anterior interosseous 4) radial 5) ulnar

Preferred Response: 4

Discussion:

Both Zaidenberg et. al. and Sheetz et. al. describe a branch of theradial artery that provides intraosseous blood supply to thedorsoradial aspect of the distal radius. Sheetz et. al. describe it asthe 1, 2 intercompartmental supraretinacular artery (I, 2 IC SRA)and Zaidenberg et. al. as the first ascending irrigating branch ofthe radial artery that arises at the radiocarpal joint, passes deep tothe first dorsal compartment, and then goes retrograde to lie onthe periosteum between the first and second dorsal compartmentswhere it sends nutrient vessels to the radius. Sheetz et. al. foundthat the 1, 2 IC SRA arose 48 mm proximal to the radiocarpaljoint, went superficial to the retinaculum between the first andsecond dorsal compartments and then went distally deep to theextensor tendons to joint either the radial artery or the dorsalradiocarpal arch. Both authors describe a pedicled bone graft thatcan be used in the management of scaphoid nonunions based onthis branch of the radial artery.

13. A seven-year-old spastic diplegic has the deformity picturedin Figure 4. He cannot actively extend the wrist but can easily bepassively corrected to 20° of dorsiflexion. He has poor grasprelease using a tenodesis effect with the wrist flexed and cannot

Page 9: ASSH Hand 1998

release with the wrist extended beyond neutral. The patient canvoluntarily flex the fingers but the first ray is fixed in anadduction contracture and the IP joint of the thumb does notactively extend. The recommended surgical plan would involve:

1) wrist arthrodesis in neutral and palmaris longus transfer to theEPL 2) FCU to ECRB tendon transfer 3) adductor policis brevis release with reverse radial forearm flapfor the first web space and FCR to EDC transfer 4) adductor pollicis brevis release, PL to EPL transfer, FCU toECRB, PT to EDC 5) proximal row carpectomy, adductor release and PL to EPL

transfer

Preferred Response: 4

Discussion:

The clinical presentation of this child demonstrates threeproblems: wrist flexion, inadequate release and a"thumb-in-palm" deformity characterized by a fixed adductioncontracture of the first metacarpal and a weak EPL. Wristarthrodesis is usually reserved for severe deformities in adultpatients where hygiene is of concern and there is little function.This patient's wrist can be passively corrected beyond neutral andhe relies upon a tenodesis effect to assist release. A fused wristwould compromise this function. Proximal row carpectomy, asdescribed by Omer, is also reserved for severe cases of wristflexion and is not required in this patient who is passivelycorrectable to neutral. The Green transfer of the FCU to the

Page 10: ASSH Hand 1998

ECRB is the classic transfer described for wrist flexion and weakgrasp. While appropriate for this patient's wrist flexion deformity,it alone will worsen the release problem and not address thethumb-in-palm deformity. This transfer is assisted by theimproved finger extension from the pronator teres to EDCtransfer. Treatment of the first ray abduction contracture involvesrelease of the adductor and possibly the flexor pollicis brevismuscles. A reverse radial forearm flap is not required.Transferring the palmaris longus tendon to the weak EPL willaugment interphalangeal extension .

14. A 39-year-old white male jams his long finger while playingbasketball. Radiographs of his injured finger are shown inFigures 5A and 5B. Recommended treatment is:

1) Extension block splinting with intermittent active motion. 2) Open reduction internal fixation. 3) Splint for three weeks, then buddy tape. 4) Volar plate arthroplasty. 5) Dynamic skeletal traction.

Preferred Response: 5

Discussion:

Dynamic skeletal traction is favored in this case because ofinvolvement of the volar 50% of the middle phalanx articularsurface as well as the dorsolateral quadrant (implied byangulation in the PA X-ray). This puts the fracture more in thecategory of a pilon fracture that Stern and coworkers say is moresafely treated with dynamic traction. Splinting alone will notprevent progressive collapse due to longitudinal force. Because ofthe asymmetrical nature of the articular surface fracture, volarplate arthroplasty would not be suitable. Open reduction andinternal fixation is technically difficult, carries additionalmorbidity, and would probably require external fixation toneutralize longitudinal forces.

Page 11: ASSH Hand 1998

15. In a skeletally immature individual, growth abnormalitiescaused by frostbite are a consequence of injury to:

1) vascular endothelium 2) secondary spongiosa 3) chondrocytes 4) osteocytes 5) perichondrial ring of LaCroix

Preferred Response: 3

Discussion:

The major lesion responsible for growth abnormalities in thejuvenile hand is direct injury to chondrocytes in the growth plateresulting in premature physeal closure. Direct injury to vascularendothelium also occurs; however, it is not a major mechanismfor skeletal abnormalities in the skeletally immature. Vascularinjury is more responsible for soft-tissue necrosis. Injury toosseous cells does occur; however, new cells are recruited andfunction in a normal fashion.

16. Which of the following procedures is most appropriate forpermanent ablation of intrinsic function in a patient with spasticparalysis of the hand and upper extremity after closed headinjury?

1) Ulnar nerve neurectomy in the forearm. 2) Ulnar nerve neurectomy in Guyon's canal. 3) Neurectomy of the motor branch of the ulnar nerve in Guyon'scanal. 4) Neurectomy of the motor branch of the ulnar nerve to theadductor pollicis. 5) Phenol block of the motor branch of the ulnar nerve inGuyon's canal.

Preferred Response: 3

Discussion:

Spastic paralysis of the intrinsic muscles of the hand limitsmetacarpophalangeal joint extension, thumb extension, andanteposition. Complete ulnar nerve neurectomy in the forearm orin Guyon's canal will dennervate the intrinsic muscles, but willalso result in an unacceptable loss of sensation. Neurectomy of

Page 12: ASSH Hand 1998

the motor branch of the ulnar nerve to the adductor pollicis willnot dennervate the interosseous or hypothenar muscles. Phenolblock of the ulnar nerve in Guyon's canal may be used as atemporizing procedure when multiple procedures are planned andthe final muscle balance is uncertain. It may also be used toprevent contractures and aid splinting when further neurologicrecovery may still occur.

17. A 59-year-old patient has a mucous cyst on the dorsum of hisindex finger at the ulnar side of the distal interphalangeal jointnear the nail matrix. The nail has a longitudinal groove adjacent tothe cyst. During surgical excision which of the followingmaneuvers will most effectively reduce the risk of recurrence?

1) Removal of the nail 2) Ablation of the nail bed 3) Capsular excision 4) Osteophyte excision 5) En bloc excision of the cyst and overlying skin

Preferred Response: 4

Discussion:

Without excision of the osteophyte underlying the osteophyte itwill recur in most, if not all, of the cases. The cyst is not alwaysseen on X-ray, but is present and underlies almost every truemucous cyst. Oblique X-rays are often helpful in confirming itspresence prior to surgery.

18. The most common hand deformity in juvenile rheumatoidarthritis is:

1) Loss of wrist flexion 2) Radial deviation of the digits at MPJ 3) Synovitis and laxity of thumb joint 4) Ulnar deviation of the carpus 5) Loss of flexion at MPJ and PIPJ

Preferred Response: 1

Discussion:

The classic early wrist deformity in juvenile rheumatoid arthritisis loss of wrist extension with a progressive flexion deformity,

Page 13: ASSH Hand 1998

emphasizing the need for wrist extension splinting as these jointsusually fuse spontaneously. Young patients with JRA (less thanfour years of age) show radial deviation of the wrist, ulnardeviation of the MPJ, and flexion deformities of the fingers. Byage six it shifts to the classic JRA presentation of ulnar deviationof the carpus and metacarpal radial deviation of the digits at MPJand loss of digit flexion at MPJ and PIPJ. The thumb showsfrequent involvement with synovitis, ligament laxity, and jointerosions of the CMC, MP, and IP joints.

19. A 26-year-old woman, under custodial care, presents to thehand clinic with a five-week history of pain and swelling in herindex finger. Nursing home records indicate a history of traumabut no additional history is available. X-rays ( Figures 6A and6B) are obtained. You recommend:

1) curettage, isoniazid, and rifampin 2) index-ray amputation 3) curettage and cephalosporin 4) biopsy and radiation/chemotherapy

5) curettage and bone graft

Preferred Response: 3

Discussion:

This patient has osteomyelitis. The X-rays suggest that she had aprevious fracture as evidenced by pin tracks. The radiographichallmarks of osteomyelitis include bone formation andresorption. In this case, resorption is evidenced by the tworadiolucencies. Bone formation is evidenced by periostealreaction and a sequestrum (density) seen in the distal lucency. Inaddition there often is soft-tissue swelling. Osteomyelitis in thetubular bones of the hand occurs most commonly after open

Page 14: ASSH Hand 1998

fractures ( I % to 11 %) and in fractures that are operativelyfixeM. Direct extension from a soft-tissue infection can alsocause osteomyelitis. S. aureus is the most common organism.Granulomatous infections of bone are rare and usually occur inimmunocompromised patients. A primary or metastaticmalignancy is rare in this age group.

20. When calcium gluconate is used to treat a hydrofluoric acidburn, its mechanism of action is:

1) local vasodilatation resulting in dilution of tissue acid levels 2) formation of insoluble complexes of calcium and fluoride ions3) bind hydrogen ions halting tissue necrosis 4) to prevent free radical formation 5) increased calcium levels that buffer the acidic environment

Preferred Response: 2

Discussion:

Hydrofluoric acid is an inorganic acid commonly used as anindustrial solvent in glass and metal cleaning and is usedextensively in the petroleum industry. Cutaneous exposure causesevere burns characterized by intense pain. The fluoride ionreadily penetrates the skin and binds with tissue calcium. This inturn causes a release of intra-cellular potassium that results indisruption of normal cellular processes. The increased potassiumcauses intense nerve stimulation leading to extraordinary pain.Calcium gluconate binds free fluoride ion and forms an insolublecomplex that prevents the binding of tissue calcium. This in turnprevents the depletion of tissue calcium and the secondary releaseof intra-cellular potassium.

21. A 65-year-old female sustains a fracture/dislocation of herright elbow. Three months later she complains of persistentelbow stiffness and is unable to reach her mouth with her hand.Examination reveals 50° to 80° of active motion with 80° ofpronation and 70° of supination 30°. A lateral X-ray is seen inFigure 7. Recommended treatment is:

1) Immediate excision of HO, capsulectomy, low-dose radiation. 2) Bone scan and alkaline phosphate. 3) Indocin, low-dose radiation, physical therapy. 4) Excision of HO, capsulectomy, low-dose radiation in threemonths. 5) Physical therapy.

Page 15: ASSH Hand 1998

Preferred Response: 4

Discussion:

This patient is disabled enough with elbow stiffness that limitsself-care activities about her face and head. Surgical interventionto increase range of motion is definitely indicated. Timing is thecritical issue with regard to heterotopic ossification. It is generallyaccepted that ablation of disabling HO about the upper extremityin the burned or brain-injured patient can safely be carried outafter one year. This somewhat arbitrary period of delay factors in"maturity" of the lesion as well as potential neurologic recoveryin the head-injured patient or local stability in the burn victim.Jupiter advocates earlier intervention on the patient without neuralaxis trauma. In the absence neural axis trauma and/or burns,Hastings and Graham advise deferral of surgical interventionuntil six months have elapsed from initial trauma. Bone scan andalkaline phosphate have been advocated as indicators of thematurity of the lesion but are not always dependable.

22. Myofibroblasts are the predominant cell type in:

1) Dupuytren's nodules and healing epidermal wounds. 2) Dupuytren's bands and full-thickness healing open wounds. 3) Dupuytren's nodules and full-thickness healing open wounds. 4) Dupuytren's cords and healing epidermal wounds. 5) Epibolic cell migration and healing epidermal wounds

Preferred Response: 3

Discussion:

The myofibroblast has been studied in Dupuytren's contracture.In order to detect the myofibroblast, electron microscopes are

Page 16: ASSH Hand 1998

used. Such studies have demonstrated the myofibroblast to bepresent in the nodular tissue of Dupuytren's contracture but notin the cord-like structures. Myofibroblasts have been found in afull-thickness wound between the 10th and 20th days afterwounding. This time period also coincides with maximum speedof wound contraction. Epibolic cell migration is found inepidermal wound repair. Epithelial cells migrate from the edge ofthe wound, populating the wound surface, forming a monolayerof cells at the leading edge.

23. The nerves most at risk for injury during surgical release ofthe first dorsal compartment are:

1) Dorsal sensory radial and posterior interosseous 2) Lateral antebrachial cutaneous and palmar cutaneous 3) Palmar cutaneous and dorsal sensory radial 4) Lateral antebrachial cutaneous and posterior interosseous 5) Lateral antebrachial cutaneous and dorsal sensory radial

Preferred Response: 5

Discussion:

The lateral antebrachial cutaneous and dorsal sensory radialnerves merge and intermingle over the first dorsal compartment.The posterior interosseous nerve is located between the third andfourth dorsal compartments. The palmar cutaneous nerve islocated between the flexor carpi radialis and the palmaris longus.

24. A patient has a release of the Al pulley of the thumb. Theadjacent structure most in jeopardy of operative injury is:

1) Abductor pollicis brevis 2) Ulnar digital nerve 3) Adductor pollicis 4) Flexor pollicis longus 5) Radial digital nerve

Preferred Response: 5

Discussion:

The radial digital nerve crosses the flexor tendon sheath justproximal to the Al pulley and just underneath the skin. It is the

Page 17: ASSH Hand 1998

anatomic structure most in danger of iatrogenic injury during thisprocedure.

25. A 19-year-old college student sustained a closed transversefracture of the mid-shaft of the proximal phalanx of his middlefinger. Radiographs reveal apex volar angulation of 40°. A closedreduction is performed and a functional cast brace applied. Oneweek later clinical examination and radiographs reveal completerecurrence of the initial deformity. In the operating room, you areagain able to achieve an excellent closed reduction of the fracture.You would now recommend fixation with:

1) Two percutaneous transverse Kirschner wires 2) Two percutaneous intramedullary Kirschner wires 3) Percutaneous screws 4) Dorsal mini- or micro-plate 5) Lateral mini- or micro-plate

Preferred Response: 2

Discussion:

Percutaneous intramedullary Kirschner wires or their equivalentshave set the standard among the available choices for treatment ofclosed reducible transverse diaphyseal proximal phalangealfractures. Transverse Kirschner wires are not appropriate for atransverse fracture pattern. Since the fracture can be reduced byclosed manipulation, the additional risks of an open procedure arenot justified.

26. A 25-year-old right-handed accountant suffers dorsal fracturedislocation of the PIP joint of the right middle finger. Closedreduction is easily achieved but the joint subluxes with extensionbeyond 25°. It is stable and congruent from 25° to 95° of flexion.Appropriate management includes:

1) static splinting in 30° of flexion for three weeks. 2) early ROM exercises in a 30° extension block splint. 3) an Eaton volar plate arthroplasty. 4) open reductions and internal fixation. 5) percutaneous pinning in 25° of flexion for three weeks.

Preferred Response: 2

Discussion:

Page 18: ASSH Hand 1998

Elevation is always essential following significant PIP joint injuryto avoid swelling and minimize stiffness. Splinting the joint in30° of flexion for three weeks will almost certainly result in aflexion contracture and it can take many months to regain motion.Eaton volar plate arthroplasty is indicated when accuratecongruous reduction can not be obtained. Use of the extensionblock splint (figure-of-eight type works well) will allow thepatient to regain flexion while healing occurs. A probable flexioncontracture of 30° or less is acceptable from a functional point ofview but greater than this is not. Therefore you don't want to usethis technique if 40° to 45° of flexion is needed to keep the jointreduced. The advantage of pinning the joint is that the reductioncan be maintained however, exercises cannot be started.

27. Late sequelae of frostbite may include which of the followingfindings?

1) Anesthetic digits 2) Dry skin 3) Decreased nail and hair growth 4) Increased digital warmth 5) Raynaud's phenomenon

Preferred Response: 5

Discussion:

Late sequelae of frostbite include vasospastic syndromessecondary to increases sympathetic tone. Symptoms and signsinclude heightened pain upon cold exposure, hyperhidrosis,trophic changes (increased nail and hair growth) and Raynaud'sphenomenon.

28. Which of the following is true with regard to the serratusanterior muscle free flap?

1) It is innervated by the serratus branch of the thoracodorsalnerve. 2) It may be raised as a muscluocutaneous flap. 3) It can be used as a functional muscle transfer. 4) Its use is limited by a short (6 cm) vascular pedicle. 5) The donor site usually requires skin grafting.

Preferred Response: 3

Page 19: ASSH Hand 1998

Discussion:

The serratus anterior muscle may be transferred as a free-flap forsmall upper extremity defects. The flap has a long vascularpedicle that can be lengthened to 15 centimeters by dividing thethoracodorsal artery and vein and dissecting the subscapuloarterial system in the axilla. The serratus anterior muscle isinnervated by the long thoracic nerve. It can be used as afunctional muscle transfer for the upper extremity or hand. Thecutaneous territory of the serratus flap has not been defined. Thedonor site is always closed primarily.

29. A 30-year-old male presents with the abnormality seen inFigure 8. It has gradually enlarged over 10 years, has beenintermittently painful and has ulcerated on several occasions. Thephysician should recommend:

1) Angiographic evaluation with gelfoam and irradiation. 2) Ray resection. 3) Resection of all abnormal vessels, and vein grafting of thedigital artery. 4) Combined embolization and surgical resection. 5) Observation.

Preferred Response: 5

Discussion:

This is a low flow venous malformation. Although most continue

Page 20: ASSH Hand 1998

to enlarge and cause a variety of symptoms including pain,ulceration, and limited motion surgical treatment is recommendedonly in certain instances. Recurrence is probable followingresection, and embolization and irradiation are of no benefit. Thispatient's symptoms are relatively mild and treatment is notnecessary.

30. A 35-year-old skier falls and injures her wrist. She cannotremember the exact mechanism of injury. The scaphoid andlunate are flexed and the triquetrum is extended on a standardlateral radiograph. These findings are most consistent with:

1) Scapholunate dissociation 2) Perilunate dislocation 3) Lunate dislocation 4) Lunotriquetrial dissociation 5) Mid-carpal dislocation

Preferred Response: 4

Discussion:

Scaphoid and lunate flexion and the triquetral extension clearlydefine the carpal malalignment that occurs with a completetriquetral-lunate ligament tear and consequent dissociation.

31. Prolonged pressure on a peripheral nerve leads todemyelination, usually followed by remyelination. There isresultant slowing of nerve conduction over the involved segment.This is due to:

1) Impaired axoplasmic transport. 2) Shorter internodal distances with remyelination. 3) Thinner layer of myelin. 4) Endoneurial edema. 5) Intussusception of successive nodes.

Preferred Response: 2

Discussion:

Demyelination occurs when prolonged pressure on a nerve leadsto intussusception of successive nodes of Ranvier. When themyelin sheath reforms, the internodal distances are decreased (i.e.the nodes are closer together). Since saltatory conduction occursbetween nodes a shorter distance between successive nodes will

Page 21: ASSH Hand 1998

lead to a reduction in conduction velocity.

32. A 60-year-old male complains of wrist pain and weakness ofgrip eight months after a resection of the distal ulna with ECRLto ECU tendon transfer. Pre- and postoperative radiographs areshown in Figure 9A and Figure 9B. This sequence of events isprimarily due to:

1) Resection of the distal ulna 2) Attenuation of the radiocarpal ligaments 3) Attenuation of ulnocarpal ligaments 4) ECRL to ECU tendon transfer 5) Rupture of TFC

Preferred Response: 2

Discussion:

Contrary to previous teaching, ulnar translation of the carpus isprimarily due to radiocarpal ligament attenuation. In this case, theinflammatory arthritis caused progressive ligamentous laxity andattenuation with loss of articular cartilage that then led to ulnartranslocation of the carpus. Had this potential for collapse beenrecognized by the surgeon at the time of distal ulnar resection, asimultaneous radiolunate fusion would have prevented the ulnartranslocation.

33. Which of the following is the most appropriate treatment fora four-year-child who presents with a Wassel Type IV thumbduplication with similar appearing duplicate thumbs?

1) Retention of the radial-most digit with ulnar collateral ligamentreconstruction. 2) Retention of the ulnar-most digit with radial collateral ligamentreconstruction.

Page 22: ASSH Hand 1998

3) The Bilhaut-Cloquet sharing procedure. 4) Observation until skeletal maturity followed by retention of theulnar-most digit with radial collateral ligament reconstruction. 5) Resection and "on-top" pollicization.

Preferred Response: 2

Discussion:

A Wassel Type IV thumb duplication consists of duplicationdistal to the metacarpal level. If the digits are similar in bony andsoft-tissue structure, the ulnar most digit is most commonlyretained secondary to the importance of a stable ulnar collateralligament. Retention of the radial most digit is more hypoplastic.The Bilhaut-Cloquet sharing procedure is more appropriate inWassel Type I (bifid distal phalanx) or Type Wassel Type II(duplicate distal phalanx) deformities. There is no reason to waituntil skeletal maturity to reconstruct a duplicate thumb.

34. Which of the following is true regarding phosphorus burnsof the hand?

1) Phosphorus ignites spontaneously when exposed to water. 2) Phosphorus may be found in insecticides and fertillzers. 3) Emergency treatment includes the application of calciumgluconate gel. 4) Surgical debridement is not recommended. 5) Acute washing with copper sulfate represents definitivetreatment.

Preferred Response: 2

Discussion:

Although phosphorus burns are more common in militarypersonnel, they do occur in civilians usually from the phosphorusin fireworks, insecticides, rodent poisons, and fertilizers.Phosphorus ignites spontaneously when exposed to air and israpidly oxidized to phosphorus pentoxide. It is extinguished bywater, but may reignite upon drying. Emergency treatmentconsists of copious water irrigation and debridement of visibleparticles. The wound should be briefly washed with 1% coppersulfate to form black cupric phosphide and facilitate the removalof phosphorus particles. Copper sulfate is not a form of treatmentand should be washed off to prevent copper toxicity after all

Page 23: ASSH Hand 1998

particles have been removed. Calcium gluconate is used in thetreatment of burns caused by hydrofluoric acid, not phosphorus.

35. What is the effect of dextran on the coagulation system.

1) Interferes with platelet aggregation. 2) Increases the activity of antithrombin III. 3) Catalyzes the conversion of plasminogen to plasmin. 4) Serves as a plasma expander. 5) Interferes with Vitamin K.

Preferred Response: 4

Discussion:

Dextran is a partially hydrolyzed polymer obtained from thebacterium Leuconostoc mesenteroides. It serves as a plasmaexpander and increases colloid osmotic pressure. It may bedangerous to patients with CHF or decreased renal function.Dextran has no in vitro effect on platelet function. Heparin bindsto antithrombin III and increases its activity 1000 fold. Coumadininterferes with Vitamin K metabolism.

36. A 39-year-old cook presents with a volar dislocating distalradioulnar joint after a fall. A CT scan demonstrates no bonyinjury to the distal radioulnar joint. The injury most likelyresponsible for the DRUJ instability is a tear of the:

1) Extensor carpi ulnaris tendon sheath. 2) Triangular fibrocartilage disk. 3) Triangular fibrocartilage from its ulnar attachment. 4) Triangular fibrocartilage from its radial attachment. 5) Ulnocarpal ligaments.

Preferred Response: 3

Discussion:

Although several structures can be injured, a tear of the triangularfibrocartilage from its ulnar attachment is the principle lesionassociated with acute traumatic distal radioulnar joint instability.With increasing severity of injury, there is concomitant injury tothe adjacent extensor carpi ulnaris tendon subsheath, theulnocarpal ligaments, the lunotriquetral interosseous ligament,and the triquetral-capitate and triquetral-hamate ligaments.

Page 24: ASSH Hand 1998

37. Which treatment is appropriate for the reconditioning phaseof a patient suffering from lateral epicondylitis?

1) Maintenance program of stretching, eccentric strengthening,and ice massage. 2) Modalities of ultrasound, phonophoresis, and iontophoresis (Iml Lidocaine, 2 ml dexamethazone). 3) Daily static splinting and anti-inflammatory medicatlon. 4) Open revision of dorsal extensor forearm musculature,common origin, and associated aponeuroses. 5) Application of tennis elbow strap.

Preferred Response: 1

Discussion:

Three specific treatment phases have been described for lateralepicondylitis. Phase I involves reduction of inflammation, painmodulation, and release of any underlying adhesions in fasciaand trigger points, and increasing flexibility. Phase 2 consists ofinitiating eccentric strengthening exercises using one- totwopound weights, increasing to three- to five-pound weights, 10to 30 repetitions. The objective of this phase is to preventrecurrence of progressive overload of extensor tendons. The thirdphase focuses on reconditioning and functional return to priorlevels of activity. Functional strengthening programs should beclosely monitored and upgraded accordingly utilizing a worksimulator. In this phase, the importance of stretching, eccentricstrengthening, and ice massage cannot be overemphasized.Electrical modalities such as ultrasound, phonophoresis, andiontophoresis should be trialed during Phase I for six to eightapplications before evaluating a particular modality's contributionto healing the injured tissue. Indications for surgery as atreatment for lateral epicondylitis coincide with between 3% and8% of all reported cases where symptomatology persists for aduration of at lease six months with failed conservative treatmentand absence of cervical or intra-articular signs or symptoms. Acounter force tennis elbow strap may be used during Phase 3 asan adjunctive treatment to stretching, eccentric stretching, icemassage, and specific instructions in maintenance program.

38. A 27-year-old female complains of difficulty gripping eightmonths following primary repair of a tidy laceration of the flexortendons in Zone 2 of the ring finger. Specific active ranges are as

Page 25: ASSH Hand 1998

follows (with passive ranges noted in parentheses): MP - 0/90;PIP - 0/60 (100); DIP - 0/40 (70). Her finger motion has notchanged for three months despite intensive physical therapy.Recommended treatment ls:

1) Dorsal IPJ capsulotomies. 2) Flexor tenolysis. 3) Staged flexor tendon reconstruction. 4) Continued occupational therapy. 5) Extensor tenolysis.

Preferred Response: 2

Discussion:

The limited active flexion with full passive flexion indicates flexortendon adhesions following tendon repair. Since it is more thanthree to six months postrepair and a plateau has been reachedwith regard to recovery of active flexion, flexor tenolysis isindicated to restore greater active motion. Continued therapy isunlikely to help. Since she has good passive flexion, dorsalcapsulotomy is unnecessary and there is no indication forextensor tenolysis. Staged flexor tendon reconstruction might bea consideration for the salvage of a severely damaged flexortendon system, but not the primary treatment for this supplefinger with intact, but limited, flexor tendon pull through.

39. A 60-year-old right-handed female has Stage IV osteoarthritisof her right thumb CMC joint, an adduction contracture of thefirst metacarpal, and a 40° hyperextension deformity of herthumb MP joint. In addition to a thumb CMC joint arthroplasty,treatment should include:

1) Adductor pollicis muscle release 2) First web release by Z-plasty 3) Flexor pollicis longus Z-lengthening 4) MP joint stabilization 5) IP joint fusion

Preferred Response: 4

Discussion:

Hyperextension of the MP joint can occur as thumb CMCarthritis progresses. This occurs as the result of progressive MP

Page 26: ASSH Hand 1998

joint capsular laxity and the unopposed extension force by theextensor pollicis brevis. It is aggravated as the first metacarpalbecomes fixed in the flexed and adducted position withprogression of the thumb CMC arthritis. The MP hyperextensiondeformity increases the stress on the CMC joint, and may causethe CMC arthroplasty to fail if it is not corrected. Fusion of theMP joint in 15° to 25° of flexion will permanently stabilize theMP joint, protect the CMC joint arthroplasty, and interfereminimally with thumb function. Soft-tissue stabilization may beadequate for less severe MP joint hyperextension .

40. Initial postoperative therapy following repair of a Zone IIflexor tendon in a six-year-old boy should include:

1) Kleinert early passive mobilization. 2) Early active place/hold mobilization. 3) Early active mobilization. 4) Immobilization. 5) Duran/Houser early passive mobilization.

Preferred Response: 4

Discussion:

Immobilization would be the best choice in treating this case.Such a technique is still the treatment of choice for patients under10 years of age, those with cognitive deficits, or those wherecompliance/participation is questionable. Immobilization isnecessary to protect the repair in such populations or to protectother structures surrounding the repair. The immobilization phasecontinues from the time of repair until three to four weekspostoperatively, after which mobilization techniques are initiated.Both the Kleinert and the Duran/Houser early passivemobilization protocols are appropriate treatment options in areliable adult patient. The patient must demonstrate excellentunderstanding and awareness of the rationale and must employexcellent independent technique to prevent tendon ruptures. Earlyactive mobilization and early active place/hold mobilizationtechniques are also appropriate for recently injured tendons.However, similarly, such treatment would not be applicable in asix-year-old boy.

41. A 66-year-old female suffered an intra-articular fracture ofher right distal radius with much dorsal comminution. The junior

Page 27: ASSH Hand 1998

orthopaedic resident performed an initial reduction andre-established anatomic alignment. She is placed in a long-armcast with good molding that extends from the digital crease toabove the elbow. She was sent home with a sling and told toexercise her fingers. The most likely long-term complication ofthis treatment if not modified promptly is:

1) limited MCP flexion. 2) painful flexion and extension of the wrist. 3) cast sores 4) limited pronation and supination of the forearm. 5) carpal tunnel syndrome.

Preferred Response: 1

Discussion:

As long as the reduction remains anatomic, no additionalimmediate attention needs to be paid to the fracture. However, inthis type of severe fracture, with elevation limited by the sling,swelling is likely to occur. The MCP joints are being held in fullextension by the cast which extends to the mid proximal phalanx(MP flexion crease), it should extend only to the distal palmarcrease. This combination of swelling and MCP jointsimmobilized in extension will commonly lead to MCP jointextension contractures with severe functional consequences.Although carpal tunnel syndrome may occur following distalradius fractures, nothing in the treatment described above wouldbe more likely to cause it.

42. A 20-year-old graduate student caught her thumb in acentrifuge and complains that it went "out of place." Examinationreveals a hyperextension deformity of the MCP joint, flexion atthe IP joint, intact skin, and no neurovascular deficits.Radiographs of her thumb are shown in Figure 10. Treatmentshould consist of:

1) Closed reduction by distraction, hyperextension, and thenflexion of the MCP joint. 2) Closed reduction by pressure over the dorsum of the base ofthe proximal phalanx, and then flexion. 3) Volar incision, intrinsic release, reduce the proximal phalanx. 4) Volar incision, retrieve the volar plate, and reduce the proximalphalanx. 5) Dorsal incision between the extensor pollicus longus andbrevis, vertical incision in the middle of the volar plate, reduce the

Page 28: ASSH Hand 1998

proximal phalanx.

Preferred Response: 2

Discussion:

This is a simple dislocation. The volar plate is not interposedbetween the dorsum of the metacarpal head and the base of theproximal phalanx. Pressure over the base of the proximal phalanxand flexion will achieve reduction. Distraction andhyperextension will flip the volar plate between the dorsum of themetacarpal head and the base of the proximal phalanx convertinga simple into a complex MPJ dislocation. Intrinsic tightness is asecondary rather than a primary problem. Release is not indicatedfor either simple or complex dislocation. The open proceduresare reserved for complex dislocations.

43. A 53-year-old female with rheumatoid arthritis affecting bothhands for the past 10 years complains of inability to flex herthumb for two weeks. There has been no pain or snapping. Themost likely diagnosis is:

1) FPL rupture from attrition. 2) FPL rupture from direct invasion. 3) trigger thumb. 4) anterior interosseous nerve compression. 5) destruction of the MP joint.

Preferred Response: 1

Page 29: ASSH Hand 1998

Discussion:

Rheumatoid arthritis can and does produce proliferativetenosynovitis that directly invades and ruptures flexor or extensortendons, but it more commonly causes bony erosions thatproduce an attritional rupture of the tendons. There is seldom aprodome before tendon rupture in RA, unlike a trigger thumb orfinger, which usually has a period of triggering, snapping or painbefore becoming locked in flexion or extension. Anteriorinterosseous nerve compression, a relatively rare condition, canoccur in patients with RA, but the incidence would not be anygreater than for the general population.

44. Which of the following statements regarding the evaluation ofpatients with carpal tunnel syndrome is true?

1) Routine wrist radiographs should be performed becauseabnormalities are seen in 33% of patients. 2) Two-point discrimination tests will be abnormal once a fewfibers of the nerve demonstrate a conduction abnormally. 3) Distal sensory latency of greater than 2.5 mm/sec is consistentwith carpal tunnel syndrome. 4) The carpal tunnel compression test has been found to be moresensitive and specific than Tinel's or Phalen's tests. 5) The symptoms of carpal tunnel syndrome usually have anacute onset.

Preferred Response: 4

Discussion:

A recent study in patients with electrodiagnostically-proven carpaltunnel syndrome shows that the carpal compression test wasmore sensitive and specific than either the Tinel percussion test orthe Phalen wrist flexion test. Wrist radiographs are abnormal in33% of patients with carpal tunnel syndrome but have therapeuticsignificance in only .4% of cases and should not be routinelyordered. Two-point discrimination remains intact even if only afew fibers are conducting normally. Distal sensory latencygreater than 3.5 mm/sec is consistent with carpal tunnelsyndrome. The symptoms of carpal tunnel syndrome occurgradually.

45. A 20-year-old white male jams his right long finger.Examination reveals palpable tenderness and swelling about theradial side of the PIP joint. Radiographs show I mm joint space

Page 30: ASSH Hand 1998

widening on the radial side of the PIP joint, but no fractures.Lateral stress testing reveals more than 20~ angulation. Treatmentshould include:

1) Immobilize 10 days; then begin intermittent active motion withcontinued splinting. 2) Immediate range of motion; elastic wrap to control swelling. 3) Open ligament repair. 4) Transarticular K-wiring for three weeks, then active range ofmotion. 5) Buddy tape to index finger and begin immediate range ofmotion.

Preferred Response: 3

Discussion:

The apex radial angulation caused by slight joint space wideningon the radial side is suggestive of interposition ligament fibersand probable complete tear of the radial collateral ligament andradial side of the volar plate. Kieflhaber, et. al. reported thatcomplete tears of PIP joint collateral ligaments will be associatedwith more than 20° of angulation on stress testing. The lateralcollateral ligament is frequently folded into the joint and surgicalrepair is recommended. Although significant instability maysometimes heal satisfactorily, interposition of ligament tissue willnot.

46. Normal excursion of the ulnar nerve above and below theelbow is approximately:

1) 1 cm above and 3 mm to 6 mm below the elbow 2) 3 mm to 6 mm above and 1 cm below the elbow 3) 2 cm above and 2 cm below the elbow 4) 2 cm above and 1 cm below the elbow 5) 1 cm above and 2 cm below the elbow

Preferred Response: 1

Discussion:

Neural excursion and elongation are obligate properties ofperipheral nerve that provide for accommodation to the changesin length and geometry associated with normal motion. A spiralorientation of axonal and connective tissue within peripheralnerve may allow elongation of the nerve trunk to occur without

Page 31: ASSH Hand 1998

concomitant changes in the strain of individual axons, withincertain limits. The epineurium acts as an interface between theexternal environment of the nerve bed and the internalcomponents of the peripheral nerve trunk. External gliding occursbetween the nerve bed, neuromesentary, and epineurium, whileinternal gliding appears to occur between the epineurium,interstitial connective tissue, and nerve fascicles. Apfelberg andLarson noted that neuromesentary allows I cm of human ulnarnerve excursion above the elbow and 6 mm below it. Wilgis andMurphy found 9.8 mm of the ulnar nerve excursion above and3.2 mm of excursion below the elbow. The correct answer in thiscase is a range that encompassed both of these studies.

47. A 29-year-old right-handed plumber suffers a dorsaldislocation of the PIP joint of the ring finger. A congruentreduction is performed and the reduction is stable through a fullROM under anesthesia (digital block). The physician should nowrecommend:

1) Elevate for 36 hours, splint only the PIP joint in 5° of flexion;start motion exercise in seven to 10 days. 2) Elevate for seven days or more if swelling persists, splint onlyPIP joint in 45° of flexion; start motion at three weeks. 3) Elevate until acute inflammation subsides; splint the PIP joint45° of flexion; start motion exercise at seven to 10 days. 4) Elevate until acute inflammation subside; splint the PIP joint in5° of flexion; start motion exercise at seven to l0 days. 5) Elevate for 36 hours; buddy tape the ring to the middle finger;immediate motion exercise.

Preferred Response: 4

Discussion:

The most common long-term complication of stable reductions ofPIP joint dorsal dislocations is stiffness and a degree ofpermanent swelling. Swelling will be minimized if elevation iscontinued until the acute inflammatory response to the injury hassubsided, which usually takes at least seven to 10 days. Splintingthe PIP joint in 45° of flexion for three weeks will result in volarplate adhesions and a difficult to correct flexion contracture. Witha stable reduction motion in a figure-of-eight extension blocksplint can be started safely once pain is minimal, usually seventold days. Buddy taping would be acceptable but with ulnarcollateral ligament injury, should be taped to the small finger.

Page 32: ASSH Hand 1998

48. A 65-year-old male complains of decades of pain and markedstiffness of the fingers. His range of motion is shown in Figures11A and 11D. X-rays of photos are shown in Figures 11E and11F. Rheumatoid titer levels have been normal. HLA-BW17 andHLA-BW38 titers are elevated. His most likely diagnosis is:

1) psoriatic arthritis 2) gout 3) pseudogout 4) rheumatoid arthritis 5) osteoarthritis

Preferred Response: 1

Discussion:

While gout or pseudogout can occur in conjunction with psoriaticarthritis, the history of decades of unremitting symptoms makethis less likely. Clinical photos show pitting of the nails,characteristic of psoriatic arthritis, and X-rays show involvementof MCP, PIP, and DIP joints. The rheumatoid factor ispersistently negative, which argues against rheumatoid arthritis.The marked osteolysis of the DIP joint is not seen inosteoarthritis and such bone resorption is usually associated withgeneralized osteopenia in rheumatoid arthritis. This combinationof osteolysis adjacent to bone of normal density and panarticularerosions, rather than marginal erosions, is characteristic ofpsoriatic arthritis. Finally, the marked tenosynovitis and "sausagefinger" are characteristic of psoriatic arthritis.

49. The sensibility test with the highest level of inter-raterreliability is:

1) Tuning fork. 2) Two-point discrimination.

Page 33: ASSH Hand 1998

3) Ridge device. 4) Semmes-Weinstein monofilaments. 5) Ninhydrin test.

Preferred Response: 4

Discussion:

The level of variance using hand-held touch stimuli is inherentlyhigh. As testing instruments, monofilaments are unique in theirability to control the amount of force applied. These produce themost sensitive and reliable data of all clinical sensibilityassessment instruments currently available. In using two-pointdiscrimination, differences between the amount of force appliedto the one point and that applied to the two points easily exceededthe sensory threshold for normal sensation, even amongexperienced surgeons and hand therapists. This lack of forcebecomes amplified during moving two-point discrimination.Inter-rater reliability is poor. Ridge device (sensitometer)introduces tissue displacement rather than applied force.However, validity and reliability remain questionable. Vibratoryinstruments (tuning fork) similarly lack control of force duringinstrument application, therefore affecting reliability. Also, mostavailable vibration instruments are of fixed frequency that varythe intensity; they do not provide variable frequencies at fixedintensities. Ninhydrin test identifies areas of disturbance of sweatsecretion following peripheral nerve disruption. This test is notspecifically used to evaluate sensibility, it tests pseudomotorfunction.

50. Lupus arthritis in the hand is characterized by:

1) Extremely stiff interphalangeal joints 2) Erosion of subchondral bone and joint space narrowing 3) "Punched-out" bone lesions in the peri-articular areas 4) Deformities of the digits without joint destruction 5) Frequent involvement of the distal interphalangeal joints andnail pitting

Preferred Response: 4

Discussion:

Systemic lupus erythematosus is more prevalent in young womenand in the hand causes joint deformities as the result of ligamentand volar plate laxity and tendon subluxation that leads to joint

Page 34: ASSH Hand 1998

imbalance without the erosive destruction of articular cartilageseen in other forms of arthritis. Psoriatic arthritis results inextremely stiff joints frequently involving the DIP joints and iscommonly accompanied by nail pitting. Rheumatoid arthritiscauses bone erosion and joint space narrowing, while gout cancause circumscribed bone erosion or peri-articular "punched-out"lesions.

51. Which of the following is the most accurate description of thelocal tissue response to a tissue expander?

1) Epidermis thins. 2) Dermis thins. 3) Subcutaneous tissue hypertrophies. 4) Adjacent muscle enlarges. 5) Local vascularity decreases.

Preferred Response: 2

Discussion:

The dermis shows a decrease in thickness following insertion ofa tissue expander. This thinning is not related to the volume ofexpansion, the anatomic location, or the rate expansion. Thecollagen bundles are noted in a characteristic orderly and parallelalignment. The epidermis initially increases in thickness. Thisinitial thickening will gradually return to original baseline valueswith further expansion. Tissue expansion causes significantatrophy of muscle, as well as thinning of the subcutaneousadipose tissue. Expanded tissue shows an increase in vascularity. 52. A 64-year-old woman presents with a four-month history ofsnapping and locking of the long finger with difficulty extendingthe finger at the MP joint. With flexion, the extensor tendonshifts ulnar to the MP joint and the finger locks in flexion andslight ulnar deviation at the MP joint. Once the finger is passivelyextended, alignment is restored and she can then actively maintainthe extended posture. Treatment should consist of:

1) Injection of the flexor tendon sheath. 2) Release of the ulnar intrinsic tendon. 3) Reconstruction of the radial sagittal band. 4) Release of the Al pulley. 5) Release of the ulnar collateral ligament.

Preferred Response: 3

Page 35: ASSH Hand 1998

Discussion:

Rupture of the radial sagittal band results in ulnar subluxation ofthe extensor mechanism at the MP Joint with snapping and theposture noted in this patient. If seen acutely, this injury is treatedby splinting the MP joint in extension. When seen late, it is bestmanaged by reconstruction of the radial sagittal band. Thefindings are distinctly different from those seen in trigger fingers,in which the sagittal alignment of the finger is not altered and theinterphalangeal joint locks, as opposed to the MP joint. The UCLand ulnar intrinsic do not play a role in the pathology of thisentity.

53. Which of the following statements regarding isolatedavulsion of the flexor digitorum superficialis is true?

1) Isolated rupture of the FDS is more common than isolatedrupture of the flexor digitorum profundus. 2) Isolated flexion of the digit reveals a mass at the A4 pulley. 3) Most patients display limited active proximal interphalangealjoint motion at the time of diagnosis. 4) Proximal migration of the ruptured tendon ends are invariablyfound at the level of the proximal carpal canal. 5) Rheumatoid arthritis, tenosynovitis, fractures, and tuberculosisare not related to closed tendon avulsion of FDS or FDP.

Preferred Response: 3

Discussion:

A recent series of 11 patients with isolated, closed avulsions ofthe FDS tendon reported on physical findings and the effect oftreatment. All were found to have a tender palmar mass at the Alpulley and loss of active FDS function. In addition, all had someloss of PIP ROM, many with a PIP contracture requiring surgicalrelease. Surgical exploration revealed the retracted FDS ensnaredaround FDP tendon at the Al pulley. Following tenolysis, FDSexcision and capsulotomy, when indicated, each patient achieved afunctional result. Rheumatoid arthritis, tenosynovitis, fractures,bony abnormalities and tuberculosis have all been implicated inclosed tendon rupture of the FDS or FDP.

54. A 42-year-old female cut her index finger with a clean knifeover the volar aspect of the mid proximal phalanx. She sought no

Page 36: ASSH Hand 1998

medical attention at the time of the injury. Subsequently, shenoted a painful clicking of this finger with flexion and extension.Her active and passive range of motion is normal. She is referredto you after treatment by another surgeon that included acortisone injection first and then release of the Al pulley.Treatment should consist of:

1) observation. 2) another cortisone injection. 3) debridement of the partial tendon laceration(s). 4) incision of the A2 pulley. 5) excision of the radial slip of the sublimis tendon.

Preferred Response: 3

Discussion:

This patient has a mechanical problem from a partial laceration ofa flexor tendon that is catching on the tendon sheath. Thisproblem is not caused by inflammation and therefore will notrespond to an injection. Incision of the A2 pulley will most likelycause bow stringing. If the injured tendon is debrided withoutinjury to additional annular pulleys the clicking and pain shouldresolve. Excising the sublimis tendon is not necessary.

55. A young adult sustains the closed injury shown in Figures12A and 12B. Treatment should consist of:

1) External fixation 2) Minicondylar plate 3) Two Kirschner wires 4) A single Kirschner wire 5) Dynamic splinting

Page 37: ASSH Hand 1998

Preferred Response: 3

Discussion:

Unicondylar fractures require fixation to assure healing withoutdisplacement. A single Kirschner wire will not prevent fragmentrotation while a second Kirschner wire or two mini screws will.Neither dynamic splinting nor external fixation will reliablycontrol the fracture fragment. Miniplates are not indicated for thisfracture.

56. The sympathetic nerve fibers to the ulnar artery at the level ofGuyon's canal are typically derived from:

1) The deep motor branch of the ulnar nerve. 2) The nerve of Henle'. 3) The Riche' - Cannieu anastomosis in 25% of cases. 4) Fibers from the nerve to the opponens pollicis. 5) Fibers from the common digital nerve to the fourth web space.

Preferred Response: 2

Discussion:

The nerve of Henle' first described in 1868, was studied byMcCabe and Kleinert in 1989. This branch of the ulnar nervemay arise in the mid or distal forearm and typically gives offfibers to the ulnar artery. These are thought to carry sympatheticinnervation to the ulnar artery and are of importance whenperforming local svmDathectomy with excision for ulnar arterythrombosis.

57. A 45-year-old mechanic is referred because of continued painin the volar proximal forearm and numbness in the thumb, index,long, and ring fingers, and thenar eminence despite a carpaltunnel release six months ago. The symptoms are exacerbatedwith active tool use, and are absent at night. Tinel's and Phalen'ssigns are negative at the wrist. Percussion over the proximal volarforearm is painful with paresthesias radiating to the fingers. Themost likely diagnosis is:

1) Incomplete release of the transverse carpal ligament. 2) Anterior interosseous syndrome. 3) Thoracic outlet syndrome. 4) Wartenberg's syndrome.

Page 38: ASSH Hand 1998

5) Pronator syndrome.

Preferred Response: 5

Discussion:

The predominance of forearm pain, absence of night symptoms,and thenar numbness are atypical for carpal tunnel syndrome andmore consistent with pronator syndrome, hence the absence ofrelief with carpal tunnel release. The anterior interosseous nerveis a purely motor branch of the median nerve. Compressioncauses weakness of the protundi to the index and long fingersand FPL, but no sensory symptoms. Thoracic outlet syndromeusually presents with lower trunk/medial cord (ulnar nervedistribution) symptoms. Wartenberg's syndrome refers tocompression of the superficial radial nerve at the distal forearm.

58. A 22-year-old male jams his middle finger playing basketball.Examination reveals that the DIP joint rests in 30° of flexion andthat the patient is unable to actively extend the joint from thisposition. Radiographs reveal a fracture of the dorsal lip of thedistal phalanx involving 20% of the articular surface. There is novolar subluxation of the distal phalanx. Treatment should consistof:

1) Static splinting; DIPJ in flexion; PIPJ in extension 2) Miniscrew fixation of the fragments 3) Static splinting; DIPJ in extension; PIPJ in extension 4) Tension-band wire fixation of the fragments 5) Static splinting; DIPJ in extension; PIPJ free

Preferred Response: 5

Discussion:

Static splinting of the DIPJ in extension and the PIPJ free isindicated for bony mallet finger deformities with small fracturefragments and no subluxation. Splinting the DIPJ in flexionwould accentuate the displacement of the dorsal fragment.Splinting the PIPJ in extension would risk permanent stiffness ofthat joint. Screw or tension-band wiring would have a substantialrisk of shattering such a small fragment and may contribute toDIP joint stiffness.

59. Which of the following statements is true regarding ulnar

Page 39: ASSH Hand 1998

club hand?

1) It is frequently associated with cardiac, hematopoietic, andgastrointestinal problems. 2) It is rarely associated with syndactyly. 3) It is frequently associated with elbow abnormalities. 4) Surgical treatment consists of early wrist fusion. 5) It is associated with a high familial incidence.

Preferred Response: 3

Discussion:

Ulnar club hand is far less common than radial club hand. It ismore commonly associated with other musculoskeletal anomaliesand not visceral abnormalities. Although rare familial cases havebeen reported, this occurrence is felt to be sporadic. Syndactyly ispresent in 34% of patients. In radial club hand, the elbow mayhave an associated contracture but is mechanically stable. Ulnarclub hand, however, is frequently associated with elbow instabilityand possibly radial head dislocation or radiohumeral synostosis.The wrist is stable and does not require fusion. Treatment variesand maybe directed towards resection of the ulna anlage, andradial or humeral osteotomies.

60. Severe swelling of the dorsum of the hand following injury tothe hand and wrist may produce stiffness, even of uninjured partsbecause:

1) it is so painful, the patient cannot do exercises. 2) when the dorsal skin is expanded by swelling it will splint theMCP joints in extension. 3) it may lead to infection. 4) RSD will occur next. 5) it will cause arthritis of the small joints of the hand.

Preferred Response: 2

Discussion:

Excessive swelling of the dorsum of the hand may or may not beaccompanied by pain but will always splint the MCP joints inextension with the collateral ligaments in the shortened position.This frequently leads to a permanent extension contracture. RSDis usually preceded by swelling but its cause is multifactorial and

Page 40: ASSH Hand 1998

swelling alone may not lead to it. In the absence of an openwound, swelling is not likely to be a proximate cause of infection.Swelling may cause arthrofibrosis of small joints but not arthritis.

61. A 45-year-old female complains of thenar eminencenumbness and severe dysesthesia six months following a carpaltunnel release. Exam reveals a well healed scar that obliquelycrosses the wrist flexion crease between the palmaris longus andflexor carpi radialis tendons, with marked sensitivity,Conservative care has failed to relieve her symptoms. Treatmentshould consist of:

1) Release of the flexor carpi radialis tendon sheath. 2) Scar excision with full-thickness skin graft. 3) Repeat carpal tunnel release. 4) Transposition of palmar cutaneous neuroma. 5) Repair of the recurrent motor branch of the median nerve.

Preferred Response: 4

Discussion:

The palmar cutaneous branch of the median nerve providessensation to the thenar eminence and emerges proximal to thewrist flexion crease between the palmaris longus and the flexorcarpi radialis tendons. Knowledge of its anatomy is essential toavoid injury and subsequent neuroma tormation. In the event ofsymptomatic neuroma formation, transposition can provide someimprovement. The other choices do not address the source of thepatient's problem.

62. A four-year-old girl accidentally lacerates her palm. Twoweeks after wound closure she complains of increasing pain andswelling about the wound. Examination reveals a tender pulsatilemass. The clinical appearance of her hand is shown in Figure13A and the appearance of the mass at surgical exploration inFigure 13B. The most likely diagnosis is:

1) Wound hematoma. 2) Mycotic aneurysm. 3) False aneurysm. 4) Ateriovenous malformation. 5) True aneurysm.

Page 41: ASSH Hand 1998

Preferred Response: 3

Discussion:

This is a pseudo-aneurysm that occurred from iatrogenic injuryto the ulnar artery at the time of suture. Unlike true aneurysms,these contain no arterial wall elements. Mycotic aneurysms areseen most commonly in IV drug abusers. Pseudo-aneurysms canoccur in hemophiliacs, but this is an x-linked recessive diseaseoccurring in males.

63. A 36-year-old right-handed female has a nine-month historyof deQuervain's stenosing tenosynovitis. She has failedconservative and surgical treatment is planned. A complicationunique to the recommended procedure is:

1) volar tendon subluxation. 2) infection. 3) swelling and stiffness. 4) neuroma of the radial sensory nerve. 5) Prieser's disease.

Preferred Response: 1

Discussion:

The patients diagnosis is deQuervain's disease and the propersurgical solution is incision of the first extensor tendoncompartment. Choices (2) and (3) are possible with anyoperation. While neuroma of the radial sensory nerve may occurwith this surgery, it is a risk as with any surgery in this area, i.e.basal joint arthroplasty, triscaphe arthrodesis. Volar subluxationof the tendons of this compartment can occur even if the sheath iscut on its most ulnar border and, when symptomatic, may requirepulley reconstruction. Prieser's disease is idiopathic avascular

Page 42: ASSH Hand 1998

necrosis of the scaphoid.

64. A 67-year-old male returns home after a golf and fishing tripto Florida. He complains of severe pain and swelling in his left,non-dominant wrist. There is no history of trauma or systemiccomplaints. Radiographs show a mild scapholunate dissociationand calcific stippling in the TFCC region. Treatment shouldconsist of:

1) immediate irrigation and debridement of the wrist joint. 2) splinting and anti-inflammatory medication. 3) STT fusion. 4) aspiration and culture for atypical mycobacteria. 5) oral antibiotics.

Preferred Response: 2

Discussion:

The most likely diagnosis is an acute pseudogout attack withdeposition of calcium pyrophosphate within the TFCC. Infection,either with bacteria or mycobacterium marinum is within thedifferential diagnosis but are very rare in comparison withpseudogout, which commonly affects the wrist in this age groupof patients.

65. An 18-year-old man sustains an accidental gun shot wound tothe distal forearm. His injuries include an open, segmental radiusfracture and laceration of the median nerve. Followingdebridement of the wound and stabilization of the radius fracture,he is left with a 6 cm defect in his median nerve. Which of thefollowing is true regarding treatment of the nerve defect?

1) The sural nerve can provide up to 20 cm of useful nerve graftmaterial. 2) Vascularized nerve grafting produces superior resultscompared to non-vascularized grafting. 3) Aligning the motor and sensory fascicles in the proximal anddistal stumps is an important factor in good results. 4) Using sensory re-education, S3+ sensory function has beenreported in only 30% of patients. 5) Because of the length of defect, primary repair or graftingshould not be done and tendon transfers should be done at a latertime.

Page 43: ASSH Hand 1998

Preferred Response: 3

Discussion:

Nerve defects up to 8 cm or 10 cm in length are amenable tonerve grafting. The sural nerve can provide up to 40 cm of nervegraft material. Both experimental and clinical studies onvascularized nerve grafting fails to show any advantage overnon-vascularized grafting. The ability to align proximal and distalnerve stumps in their correct orientation prior to graftingdramatically improves results. S3+ sensory function has beenreported in 80% of patients following median nerve grafting.

66. An axial flag flap raised on the dorsum of the long fingerutilizing the radial dorsal digital artery can be used to cover adefect on:

1) The volar distal palm at the based of the ring finger. 2) The dorsal proximal interphalangeal joint of the long finger. 3) The ring-long web space. 4) The index volar pad. 5) The volar distal palm at the base of the index finger.

Preferred Response: 5

Discussion:

The axial flag flap utilizes the arterial supply of the dorsal digitalartery. It is most reliably taken from the dorsal aspect of theproximal phalanx of the index or long fingers. When it is takenfrom the long finger and based radially using the radial dorsaldigital artery, it can be used to cover defects over the dorsal MPjoint of the long or index finger or the dorsal base of the indexfinger. It can also be used to cover the volar aspect of the base ofthe long or index finger or the distal palm at the base of the longand index fingers. It can reach defects on the palmar surface ofthe MP joint of the index or long fingers by carrying the flapthrough the web space. Because the described flap was basedradially, it cannot be used to reach the ring-long finger web or thevolar distal palm at the base of the ring finger. Since it is basedproximally, it cannot reach the dorsal proximal interphalangealjoint of the long finger. Based on the lack of flexion of the indexfinger, it would be virtually impossible to use it to cover the indexfinger volar pad.

Page 44: ASSH Hand 1998

67. A 70-year-old female is initially treated for a non-displaceddistal radius fracture in a short arm cast that extends to the palmardigital crease shown in Figures 14A and 14B for six weeks. Afterconcerted and exhaustive rehabilitation elforts, her range ofmotion is seen in Figures 14C and 14D. The patient reports thatthis is a serious functional limitation. Recommended treatment is:

1) dynamic splinting 2) MCP arthroplasties 3) metacarpal neck osteotomies 4) extensor tenolysis 5) dorsal capsulotomy and collateral ligament resection

Preferred Response: 5

Discussion:

The initial cast extended too far distally. The level of the MCPjoint corresponds to the distal palmar crease, not the palmardigital crease. This has allowed the collateral ligaments to tightenin extension and prevents flexion of the MCP joints. This is acommon and serious problem that is frequently not responsive toeven aggressive therapy. MCP joint arthroplasty is not requiredbecause there is nothing wrong with the articular surfaces.Metacarpal neck osteotomy would produce little gain in motionbecause the ligamentous and capsular contractions are notreleased. Thus, the best option would be dorsal capsulotomy andrelease or recession of the collateral ligaments.

68. A medical student fell on his hands while roller blading fiveyears ago and complains of bilateral, severe ulnarsided wrist pain.X-rays are shown in Figures 15A and 15B. Conservativemeasures have been exhausted and recommended treatment is:

1) TFCC repair on the left and excision of the styloid fragmenton the right 2) bilateral ulnar styloidectomy 3) left ulnar shortening diaphyseal osteotomy and right

Page 45: ASSH Hand 1998

arthroscopic TFCC repair 4) "wafer" procedure on the left and styloid fragment excision onthe right 5) Darrach procedure and ECU/FCU tenodesis stabilization onthe left and right styloid fragment excision

Preferred Response: 1

Discussion:

While ulnar styloid fracture nonunions are common, symptomsand infrequent and may be due to the fibrous nonunion orconcomitant TFCC injury. More recently, Hauck, et. al. report onthe classification and treatment of symptomatic ulnar styloidnonunion. Injuries associated with DRUJ instability were treatedby TFCC reconstruction, while nonunions with no instabilityunderwent simple excision of the fragment. This patient'sradiographs demonstrate an ulnar styloid nonunion on the rightside with a stable DRUJ (Type 1). On the left side, there appearsto be an old, healed styloid fracture, but there is residual DRUJinstability seen in the stress AP and lateral films. The correcttreatment would be left TFCC repair and excision of the fragmenton the right.

69. In fractures of the distal radius involving younger adults, the

Page 46: ASSH Hand 1998

most important prognostic indicator for long-term successfulresults is the restoration of:

1) Radial length (<3 mm shortening) 2) Articular joint congruity (<2 mm step-off) 3) Radial angulation (<15° dorsal tilt) 4) Radial shift (<3 mm) 5) Radial inclination (>10°)

Preferred Response: 2

Discussion:

Symptomatic arthritis has been shown to develop in 90% ofyounger adults who healed with residual articular surfacestep-offs of 2 mm or greater whereas it occurred in only 11 % ofthose that healed with a congruous surface. Radial length,angulation (tilt), inclination and shift also influence outcomes indecreasing order of importance.

70. A 43-year-old physician sustains a non-displaced fracture ofthe distal radius while skating. She is placed in a shortarm cast.Five weeks after injury she notes an inability to move her thumb.Treatment should consist of:

1) Corticosteroid injection at the Al pulley. 2) Short-arm spica cast. 3) IP joint arthrodesis. 4) Primary repair of the EPL. 5) EIP to EPL transfer.

Preferred Response: 5

Discussion:

A trigger thumb can cause an inability to move the thumb and canoccur during immobilization but it is usually associated with volarthumb discomfort. Other possibilities in the differential diagnosisinclude compression of the posterior interosseous nerve, but themost common possibility is rupture of the extensor pollicuslongus. A primary repair of the EPL is seldom possible after theattritional rupture of this tendon after distal radius fracture.Therefore, a reconstruction is usually the only option and caninclude a free tendon graft or EIP to EPL transfer.

Page 47: ASSH Hand 1998

71. The parents of a three month-old infant bring the child toyour office for evaluation of the right thumb. They have noticedthat he holds the digit flexed at the IP joint, and has done so forthe past month. On examination, the IP joint extends to 30°passively, with a mechanical block to further extension. Theremainder of the hand examination is normal. Recommendedtreatment is:

1) EIP to EPL tendon transfer. 2) Operative release of the Al pulley of the thumb within onemonth. 3) Echocardiogram and IVP. 4) Operative release of the Al pulley at the age of one year if thedeformity is still present. 5) Full-time splinting for six months.

Preferred Response: 4

Discussion:

The etiology of the congenital trigger thumb is unknown. Itoccurs as an isolated lesion and is not associated with othervisceral anomalies. While the problem is called "congenital," notall children are affected at birth. Some authors feel that 30% oftrigger thumbs resolve by the age of one year. Consequently,most recommend observation of the lesion until one year of agewhen anesthesia risk is lower and thumb structures are larger.Splinting has not been shown to be beneficial. Surgical release ofthe Al pulley is curative and can be performed after three years ofage with no residual deformity according to Skov, Bach, andHammer.

72. A 30-year-old cook lacerates the back of his hand with a largebutcher knife. The oblique direction of the cut produces anextensor Zone IV laceration of the index finger, Zone V lacerationof the long finger, and a Zone VI laceration of the ring finger.A11 tendons are repaired without difficulty. Postoperatively,which finger(s) has the highest likelihood of a good or excellentresult?

1) index 2) long 3) ring 4) index and long 5) long and ring

Page 48: ASSH Hand 1998

Preferred Response: 2

Discussion:

The distal Zones (I-IV) do significantly more poorly than themore proximal zones. Zone IV has a 43% good or excellentresult rate. Zone V, even with capsule laceration, has an 83%good or excellent result rate, and Zone VI has a 65% good orexcellent result rate.

73. In a patient with rheumatoid arthritis who has a stable, wellbalanced and painless wrist, painful subluxated MP joints of thefingers, and extensor tendon ruptures, the correct sequence of astaged reconstruction is:

1) MP joint reconstruction then extensor tendon reconstruction 2) Extensor tendon reconstruction, then MP joint reconstruction 3) MP joint reconstruction, then wrist fusion 4) Extensor tendon reconstruction, then MP joint fusion 5) Wrist arthroplasty, then extensor tendon reconstruction

Preferred Response: 1

Discussion:

In a rheumatoid patient with MP joint destruction requiringarthroplasty who also has extensor tendon ruptures, stagedreconstructive surgery often yields better results. The MP jointsurgery should be done first. Motion of the reconstructed jointscan be maintained with dynamic splinting until sufficient healinghas occurred to allow proceeding with extensor tendonreconstruction as the second stage. If the extensor tendon surgeryis done before the joint arthroplasties, the reconstructed tendonswill be unable to move the subluxated and arthritic joints throughthe range of motion necessary to maintain tendon function.

74. A 52-year-old professional string bass player fell from aladder and suffered a bilateral wrist injury. He complains oflocalized pain on the dorso-ulnar aspect of his right wrist (Figure16A), his bow hand. His left wrist (Figure 16B) is swollen andgenerally painful. He wishes to return to practicing andperforming as soon as possible. Recommended treatment is:

1) ORIF right wrist and ORIF left wrist (plate and screws) . 2) SA cast right wrist and external fixation left wrist. 3) SA cast right wrist and closed reduction and SA cast left wrist.

Page 49: ASSH Hand 1998

4) SA cast right wrist and ORIF left wrist (plate and screws). 5) SA cast right wrist and ORIF left wrist (intrafocal pins).

Preferred Response: 4

Discussion:

This patient has two special requirements early return toperformance and enough function in his left wrist to allow properhand position for fingering his instrument. His right bow handcan function with a short arm-cast which is the preferredtreatment for avulsion fracture of the triquetrum. To assure thathe can have adequate wrist flexion and early return to function,anatomic reduction and rigid internal fixation is required on theright. This can best be accomplished with either a dorsal or volarblade plate.

75. A 20-year-old student falls while cross country skiing. Histhumb is shown in Figure 17A and X-rays are shown in Figures17B and 17C. Treatment should include:

1) splinting until comfortable and early rehabilitation permittinguse as tolerated 2) surgical repair of the dorso-radial capsule and EPB insertion 3) surgical repair of the radial collateral ligament 4) repair and advancement of the APB insertion 5) repair of the radial collateral ligament and dorsoradial capsule

Page 50: ASSH Hand 1998

Preferred Response: 5

Discussion:

The clinical photo shows a bulge on the radial aspect of thethumb MCP joint and ulnar angulation of the thumb at the MCPjoint with the thumb in repose. This is due to loss of the primaryradial stabilizer of the joint, the radial collateral ligament complex.X-rays shown in Figure 17B and 17C confirm the diagnosis ofradial instability and show palmar subluxation of the proximalphalanx on the metacarpal head. While injuries to the ulnarcollateral ligament are more common than radial-sided injuries,the structure and importance of the radial complex is receivingincreasing attention. The role of the dorsal capsule and EPBinsertion, as well as the RCL, in supporting the proximal phalanxand preventing palmar subluxation of the proximal phalanx hasbeen described by Posner and Retaillard, among others. Faillarecently emphasized the importance of repairing the injuredstructures and confirmed that the injury may involve the RCL,EPB/capsule or both. Given the degree of instabilitydemonstrated in this patient, surgical repair is indicated. Themarked radial instability and palmar subluxation reflect injury toboth the RCL and EPB-dorsal capsule complex.

76. A patient sustains a closed, complex dorsal dislocation of hisindex MCP joint (Figures 18A and 18B). The anatomic structurein greatest jeopardy of iatrogenic operative injury during a volar

Page 51: ASSH Hand 1998

surgical approach to correct the problem is:

1) Lumbrical muscle. 2) Ulnar digital nerve. 3) Flexor digitorum superficialis. 4) Radial digital nerve. 5) Flexor digitorum profundus.

Preferred Response: 4

Discussion:

The radial digital nerve is tented over the region of the Al pulley.The Al pulley must be incised during a volar approach in order toextract the volar plate and reduce the dislocation. It is just beneaththe skin and is the structure most likely to be injured during thisprocedure. A dorsal approach does not place this structure at riskfor injury.

77. After nerve laceration, Wallerian degeneration:

1) occurs in the proximal stump. 2) occurs by degeneration of Schwann cells. 3) causes no change within the cell body. 4) clears degraded myelin and axoplasm from Schwann celltubes. 5) causes immediate lack of response in the distal segment duringnerve conduction studies.

Preferred Response: 4

Discussion:

Wallerian degeneration occurs within the distal segment of the

Page 52: ASSH Hand 1998

nerve after laceration. The Schwann cells and the tube they formremains intact and untouched. The contents of the Schwann celltube undergo granular degeneration and are ultimately cleared bymacrophages. The cell body remains intact but increasesproduction of growth associated protein 100 fold in response tothe injury. Until complete degeneration in the distal axons occurs,there is still capacity for electrical conduction for up to threeweeks after injury.

78. Which of the following statements is true regarding MRI forZone 11 flexor tendon injuries?

1) MRI clearly differentiates tendon ruptures from adhesionswith 100% accuracy. 2) MRI cannot clearly differentiate adhesions from tendonrupture. 3) Clinical examination for tendon rupture or adhesion is moreaccurate than MRI evaluation. 4) Ultrasonography more reliably evaluates partial tendonruptures in Zone II. 5) MRI delineates soft-tissue structures in the hand poorly.

Preferred Response: 1

Discussion:

Differentiation of tendon rupture from tendon adhesion may be adifficult clinical problem and accurate diagnosis is critical. MRIhas been shown to accurately differentiate tendon rupture fromtendon adhesion. MRI has been shown to be more accurate thanclinical examination and more accurate than ultrasonography.

79. A patient develops painful snapping of the exterior tendonover the metacarpophalangeal joint of the middle finger followingan industrial accident two months ago. The patient is now unableto fully extend at the metacarpophalangeal joint and ulnarlydeviates with active extension. The most appropriate managementof this injury includes:

1) Static splint with MCP joint in extension for four weeks;immediate mobilization of the PIP/DIP. 2) Static splint with MCP joint in 70^ flexion for four weeks;immediate mobilization of the PIP/DIP joints . 3) Cast immobilization with hand in a "safe" position for fourweeks. 4) Repair of the sagittal band and stabilization of the extensor

Page 53: ASSH Hand 1998

tendon with tendon graft. 5) Dynamic MCP joint extension splint.

Preferred Response: 4

Discussion:

The patient has developed a chronic dislocation of the extensortendon over the metacarpophalangeal joint secondary to an injuryto the radial sagittal band. Acute injuries seen within two weeksshould be treated with immobilization preventing flexion at themetacarpophalangeal joint but allowing IP joint motion. Chronicinjuries more than two weeks old require surgical repair of thedefect and stabilization of the extensor tendon.

80. A 75-year-old male with severe spastic hemiplegia two yearsfollowing a stroke, has skin maceration of his fingers and palmon the involved side, secondary to severe flexion deformities ofall fingers. Local skin hygiene is a major problem.Recommended treatment is:

1) Phenol injection. 2) Intrinsic tendon releases. 3) MP joint arthrodesis in extension. 4) Superficialis to profundus tendon release and transfer. 5) Transfer of the FCU to the finger extensors.

Preferred Response: 4

Discussion:

Spasticity following a stroke can sometimes be so severe that thefingers dig into the palm, macerating the skin and hinderinghygiene. The spastic muscles are the extrinsic finger flexors,hence intrinsic releases would not solve the problem. Transfer ofthe FCU to the finger extensors would not release or overcomethe spastic finger flexors. Phenol blocks can be helpful fortemporary relief of spasticity and may be used while healing ortraining a tendon transfer, but would not be a permanent solution.Finger arthrodesis would be more complicated than necessaryand MCP joint arthrodesis would not correct flexion deformitiesat the IP joints. Superficialis to profundus transfer effectivelylengthens the spastic extrinsic finger flexor tendons, and stillallows the patient to retain some finger control.

81. A 23-year-old roofer presents to an emergency room two

Page 54: ASSH Hand 1998

hours after falling on an outstretched wrist. There were no otherinjuries and the distal neurosensory exam is normal. Radiographsare depicted in Figures 19A and 19B. Treatment should include:

1) closed manipulation and long-arm cast immobilization 2) scaphoid internal fixation and internal/external fixation radius 3) dorsal plate fixation of the radius and short-arm thumb spica 4) bone graft and internal fixation of the scaphoid plus long-armcast 5) percutaneous pins and bone graft to radius plus bone graft toscaphoid

Preferred Response: 2

Discussion:

Concomitant fractures of the distal radius and scaphoid areuncommon. Most distal radius fractures require some flexion ifimmobilized in a cast; a position not recommended for scaphoidfractures particularly if there is comminution of the anteriorscaphoid cortex. Therefore this injury requires operativemanagement. Recent literature suggests that the scaphoid requiresinternal fixation either with pins or a screw. Management of theradius fracture must be individualized and depends on thefracture pattern, degree of combination, and bone compaction.Proubasta and Lluch recommend external fixation alone. Thiscould be risky because the scaphoid could be distracted and distalradius may not completely reduce.

82. Which of the following is a contraindication to transfer of theflexor carpi ulnaris tendon for wrist extension in a patient withcerebral palsy?

1) Intelligence quotient of 75. 2) Pure athetoid pattern.

Page 55: ASSH Hand 1998

3) Hand-to-knee placement of less than five seconds. 4) Two-point discrimination of 8 mm. 5) Pre-op EMG showing FCU active in grasp only.

Preferred Response: 2

Discussion:

Tendon transfers in patients with athetoid cerebral palsy arecontraindicated. Tendon transfer in the athetoid patient mayactually worsen the pre-op state. Two-point discrimination of 8mm indicates sufficient sensibility for good outcome. IQ of 75and hand-to-knee placement of less than five seconds are alsopredictors of good results. A muscle that is active in grasp only isin the appropriate phase for transfer to the wrist extensors.

83. A healthy 15-year-old hockey player presents to the office sixweeks after injuring his right dominant hand in a skatingaccident. He struck his hand against the boards and complainedof an occasional "snapping" sensation with intermittent difficultyin extending his middle finger. X-rays are unremarkable. Onexamination, a defect is noted dorsally over themetacarpophalangeal joint on flexion. The most likely diagnosisis:

1) Traumatic dislocation of the extensor tendon associated withrupture of the sagittal bands. 2) Traumatic rupture of the extensor digitorum communis. 3) Posttraumatic flexor tenosynovitis with locking. 4) Stenosing tenosynovitis associated with collateral ligamentinstability. 5) Rupture of the junctura tendinae.

Preferred Response: 1

Discussion:

Subluxation or dislocation of the extensor digitorum communismay occur following forceful flexion or extension injuries to thefingers. Ulnar dislocation is associated with a tear to the radialsagittal band and oblique fibers of the extensor hood. A traumaticrupture would eliminate any extension or possibility of"snapping." Posttraumatic synovitis and stenosing tenosynovitiswould not be associated with a dorsal defect which occurs whenthe extensor digitorum subluxate ulnarly. Rupture of the junctura

Page 56: ASSH Hand 1998

tendinae would not cause intermittent symptoms.

84. A 57-year-old transcriptionist complains of three months ofstiffness of the right ring finger when she tries to make a fist. It isworse in the morning. She also notes a burning discomfortacross her distal palm. She denies injury. On examination, youfind that she actively flexes to within 3 cm of the distal palmarcrease when she tries to make a fist, but you can achieve fullpassive flexion without difficulty. The most likely diagnosis is:

1) PIP joint osteoarthritis. 2) PIP joint rheumatoid arthritis. 3) MCP joint osteoarthritis. 4) tenosynovial thickening. 5) overuse syndrome.

Preferred Response: 4

Discussion:

The key here is the discrepancy between active and passiveflexion that points to the muscular tendinous system as thesource of pathology. If the pathology is in the joint the loss ofmotion will be the same passively and actively. Overusesyndrome would be less likely to involve one digit and should bebetter with rest. Stenosing tenosynovitis is obvious if there is atriggering phenomenon. Frequently, however, it may present asmorning stiffness with palmar burning pain without triggering.There will be tenderness directly over the A1 pulley. A cortisoneinjection can be both diagnostic and therapeutic.

85. The most common cause of a pseudoboutonierre deformityis:

1) rheumatoid arthritis 2) a central slip injury 3) psoriatic arthritis 4) a flexor tendon injury 5) a volar plate injury

Preferred Response: 5

Discussion:

The pseudoboutonierre deformity refers to a flexion deformity of

Page 57: ASSH Hand 1998

the PIP joint that looks similar to a boutonniere deformity, but iscaused by a volar plate injury, usually as the result of an acutehyperextension injury that is either untreated or treated bysplinting with the PIP joint in flexion. It is differentiated from aboutonniere deformity by a history of an acute or semi-acuteinjury and by the absence of DIP joint extension contracture.Treatment requires splinting to stretch the contracted volar plateor surgical release of the contracted volar structures.

86. An embolus lodging in the bifurcation of the brachial arteryusually originates from:

1) Subclavian artery. 2) Common iliac vein. 3) Pulmonary artery. 4) The heart. 5) Axillary artery.

Preferred Response: 4

Discussion:

Fifteen percent to 20% of all arterial emboli are in the upperextremity. Of these, 70% are of cardiac origin. Cardiac emboliarise from a mural thrombus that forms either after a myocardialinfarction or within a ventricular aneurysm. Emboli can also beshed from mural thrombi in the atrium in association with atrialfibrillation. Emboli of cardiac origin are often quite large, andtherefore lodge proximally, often in the bifurcation of the brachialartery.

87. Outcome scores for severe fractures of the distal radiustreated by external fixation, which are based on function, pain,motion, and grip strength are most adversely affected by:

1) Fixator pin site bone infection 2) Duration of external fixator use for less than seven weeks 3) Overdistraction (carpal height index > 0.7) 4) Residual dorsal angulation less than 15° 5) Supplemental percutaneous wire fixation

Preferred Response: 3

Discussion:

Page 58: ASSH Hand 1998

Overdistraction of the carpus (CHI>0.7) and prolonged durationof use of the external fixator (more than eight weeks) areassociated with decreased wrist motion, grip strength and ADLs.Pin site infection, residual dorsal angulation less than 15°and useof supplemental percutaneous wire fixation at the fracture site donot affect outcome scores.

88. Under digital block anesthesia, it may be possible to reduce apalmar PIPJ rotatory subluxation/dislocation by applyinglongitudinal traction with a gentle twisting maneuver to the middlephalanx with the hand in which position:

1) Wrist flexion 30°; MP joint flexion 90°; PIP joint extension 2) Wrist extension 30°; MP joint flexion 30°; PIP joint flexion60° 3) Wrist flexion 60°; MP joint extension; PIP joint extension 4) Wrist extension 30°; MP joint flexion 90°; PIP joint flexion90° 5) Wrist flexion 30°; MP joint extension; PIP joint flexion 30°

Preferred Response: 4

Discussion:

A palmar PIP joint rotatory subluxation/ dislocation results fromthe buttonholing of one condyle of the proximal phalanx througha longitudinal rent between the central slip and lateral band.Flexing both MP joint and PIP joint to 90° maximally relaxes thevolarly displaced lateral band (wrist extension further relaxes theextensor mechanism) allowing the entrapped lateral band todisengage and slip dorsally and the PIP joint to reduce whengentle traction with a rotational maneuver is applied .

89. A 45-year-old female falls on her dominant arm sustainingthe injury shown in the X-rays in Figures 20A and 20B. Aclinical photo of the medial aspect of the forearm at 24 hoursafter injury is shown in Figure 20C. A valgus stress view isshown in Figure 20D. Treatment should now consist of:

1) medial collateral ligament reconstruction or repair, ORIF of theradial head fracture 2) ORIF radial head fracture 3) radial head implant 4) closed reduction and casting 5) fasciotomy and subcutaneous ulnar nerve transposition

Page 59: ASSH Hand 1998

Preferred Response: 1

Discussion:

The X-ray shows a comminuted radial head fracture. Stress viewsdemonstrate medial opening of the joint with valgus stress andthe clinical photo shows medial ecchymosis that indicates aserious medial injury, rather than an isolated radial head fracture.Correct treatment involves not only addressing the radial headinjury, but also the ulnar collateral ligament, which is the primarymedial stabilizer of the elbow. Davidson, et. al. reportedconcomitant injury to either the UCL or interosseous membranein all patients with a comminuted radial head fracture. Failure totreat the medial side risks persistent medial instability, pain andearly degenerative changes. ORIF or replacement of the radialhead with a silastic or titanium implant confers some additionalstability, but cannot substitute for a competent UCL. The mostpernicious combination is radial head excision with UCL injury.The incidence of instability, pain and tardy ulnar neuritis is quitehigh in this scenario.

90. Which of the following surgical procedures is not commonlyperformed for spastic thumb-inpalm deformity.

1) Flexor pollicis longus z-lengthening or fractional recession. 2) Thumb CMC joint arthrodesis. 3) Recession of the thenar origin. 4) Adductor pollicis origin recession. 5) First dorsal interosseous release

Preferred Response: 2

Discussion:

All of the procedures listed above have been described as primaryor secondary procedures for the correction of spastic

Page 60: ASSH Hand 1998

thumb-in-palm deformity except for CMC joint arthrodesis.While CMC arthrodesis may place the thumb in a position ofdesired anteposition out of the plane of the palm, it has not beenwidely used for the treatment of spastic thumb-in-palmdeformity.

91. A 68-year-old female with rheumatoid arthritis has anadduction contracture of the first web space. Treatment shouldinclude:

1) Release of the adductor pollicis muscle from the thirdmetacarpal 2) Z-lengthening of the flexor pollicis longus tendon 3) CMC resection arthroplasty and Z-plasty of the web skin 4) MP joint fusion and Z-plasty of the web skin 5) Web release and dorsal transposition flap

Preferred Response: 3

Discussion:

First web space contractures in rheumatoid patients are mostcommonly the result of progressive arthritis of the thumb CMCjoints that results in a flexion and adduction deformity of the firstmetacarpal and a secondary contracture of the web space.Correction of the fixed metacarpal deformity by CMC jointarthroplasty combined with a Z-plasty of the first web skinusually are sufficient to correct the web contracture, althoughoccasionally release of the adductor aponeurosis is required.

92. Studies on the effect of the flexor tendon pulley system showthat:

1) A4 pulley excision provides the largest elficiency change. 2) A I or A5 pulley resectioning had a larger effect than A4excision. 3) A2 pulley excision produces the largest efficiency change. 4) Pulleys improve angular motion of excursion by increasing themomentum. 5) The palmar aponeurosis pulley is more effective than theAIpulley.

Preferred Response: 1

Page 61: ASSH Hand 1998

Discussion:

While cutting one of the major pulleys, A2 or A4, may result insignificant changes in efficiency, a loss of both work andexcursion efficiency was noted with the loss of A4 pulley. Al andA5 are minor pulleys and show no statistical difference in workefficiency from controls if they are cut. The palmar aponeurosismay act as a pulley and should be kept intact if possible with Alpulley release for trigger fingers.

93. A 59-year-old construction worker sustained a crush injury tothe volar surface of the middle phalanx and DIP joint of his indexfinger. The wound was repaired primarily, but the disrupted FDPtendon was not repaired. Two years later he has a 15° flexioncontracture at the DIP joint with passive flexion of 25°, a range of10/95 at the PIP joint actively, and normal MP joint motion. Yourecommend:

1) Staged flexor tendon reconstruction. 2) One stage flexor tendon graft. 3) Flexor tenolysis. 4) FDP tendon advancement. 5) No surgical treatment.

Preferred Response: 5

Discussion:

This patient has an old disruption of the flexor digitorumprofundus tendon and a stable but stiff DIP joint in a functionalposition. A prerequisite for successful outcome following flexortendon reconstruction is full passive motion of the affected joints.Therefore, flexor tendon reconstruction is not indicated in thispatient. Furthermore, he has good function of the superficialis,and late flexor tendon reconstruction through an intactsuperficialis may damage that tendon and jeopardize the overallfunction of the finger. Such reconstruction would be indicated inonly selected situations. Given his stiff but well positioned DIPJoint and well functioning superficialis, no surgery is indicated.

94. The use of an above elbow thumb spica cast for six weeksfollowed by a below elbow thumb spica cast until fracture healinghas shown what end result in the treatment of a non-displacedscaphoid fracture.

Page 62: ASSH Hand 1998

1) Decreased time to bony union 2) Increased rate of nonunion 3) Decreased rate of malunion 4) Decreased rate of avascular necrosis 5) Increased incidence of elbow stiffness

Preferred Response: 1

Discussion:

Using the above treatment protocol, the time to fracture healingaveraged 9.5 weeks vs. 12.7 weeks when only a below elbowthumb-spica cast was utilized. The rate of scaphoid nonunionalso decreased with this protocol. These differences were moreevident for proximal and mid-third fractures, distal third fracturesdid uniformly well regardless of the type of cast. Rates ofmalunion and avascular necrosis and the incidence of elbowstiffness were not influenced.

95. Which of the following inhibits contraction of Dupuytren'sfibroblasts in vitro?

1) PGE, (Prostaglandin E) 2) Angiotension 3) Serotonin 4) Lysophosphatidic acid (LPA) 5) PGF2 (Prostaglandin F2)

Preferred Response: 1

Discussion:

Pharmacological regulation of Dupuytren's fibroblast contractionhas been studied in vitro. A number of agents have been found toeither inhibit or promote contraction of these fibroblasts.Angiotension, serotonin, LPA, and PGF2 have all been shown tocause contraction. PGE2, PGEI, nifedepine, and verapamil havebeen shown to be inhibitors of Dupuytren's fibroblastcontraction.

96. A nine-year-old boy with spastic hemiplegia has a wristflexion deformity. Passive extension is excellent, but he is unableto actively extend his wrist. With the wrist passively held atneutral, he can fully extend his fingers actively. You recommend:

Page 63: ASSH Hand 1998

1) FCU to ECRB tendon transfer. 2) Wrist arthrodesis at neutral. 3) Flexor-pronator slide. 4) Fractional lengthening of finger flexors. 5) Tenotomy of the wrist flexors.

Preferred Response: 1

Discussion:

The FCU to ECRB tendon transfer helps augment wrist extensorpower while reducing flexor tone at the wrist, and has beenparticularly effective in cases where good active finger extensionis present with the wrist in a neutral position. Since the fingerflexors are not excessively tight, as indicated by the ability toextend the fingers with the wrist in neutral, there is no need torelease them. Fractional lengthening or flexor-pronator slidewould unnecessarily eliminate useful wrist motion, and wristflexor tenotomy could lead to unopposed wrist extension withsecondary extension deformity.

97. Treatment for fingernail deformities associated withganglions of the distal interphalangeal joint:

1) Debridement of DIP joint osteophytes. 2) Excision of the cyst wall only. 3) Intralesional injection of corticosteroid. 4) Occlusive otoform patches. 5) Removal of the nail plate cyst and osteophytes.

Preferred Response: 1

Discussion:

DIP joint ganglions that deform the nail and present as cysticlesions have been shown to be effectively treated by excision ofthe osteophyte alone. Excision of the overlying skin or cyst wallmay risk injury to the germinal nail matrix and is not required.Occlusive treatment and intralesional injections will not eradicatethe osteophyte deemed the most important factor in preventingrecurrence.

98. Which of the following findings distinguishes compression

Page 64: ASSH Hand 1998

neuropathy of the ulnar nerve at the elbow from that at the wrist?

1) Weakness of the first dorsal interosseous. 2) Decreased sensation in the ring and little fingers. 3) Decreased sensation in the dorso-ulnar hand. 4) Weakness of the profundus to the index finger. 5) Hypothenar muscle atrophy.

Preferred Response: 3

Discussion:

The dorsal sensory branch of the ulnar nerve innervates thedorso-ulnar hand and separates from the ulnar nerve proximal tothe wrist. Thus compression of the ulnar nerve at a moreproximal level, such as the elbow, can result in diminishedsensation on the dorsoulnar hand, whereas more distalcompression at the wrist would not. The profundus to the indexfinger is medianinnervated and thereby not affected by ulnarnerve compression. The other choices can all develop as the resultof ulnar nerve compression at either the wrist or the elbow andtherefore do not help to distinguish between these two levels.

99. When comparing intrasynovial and extrasynovial tendonautografts for flexor tendon reconstruction, the extrasynovialgraft at six weeks has:

1) better gliding 2) better angular rotation at the PIP joint 3) more adhesion formation 4) better graft survival 5) Iess ultimate load

Preferred Response: 3

Discussion:

Canine studies have demonstrated that extrasynovial flexortendon grafts heal by an in-growth of extrinsic adhesions.Intrasynovial grafts heal with minimal adhesions and have bettergraft survival which results in better gliding and PIP angularrotation at six weeks postoperatively. Extrasynovial grafts at sixweeks have greater stiffness and ultimate load probably due toadhesion formation.

100. Compression of the superficial branch of the radial nerve in

Page 65: ASSH Hand 1998

the distal forearm (Wartenberg syndrome) most commonlyoccurs between which structures?

1) Extensor carpi radialis longus and extensor carpi radialisbrevis. 2) Extensor carpi radialis longus and brachioradialis. 3) Brachioradialis and flexor carpi radialis. 4) Abductor pollicis longus and extensor pollicis brevls . 5) Abductor pollicis longus and extensor pollicis longus.

Preferred Response: 2

Discussion:

The superficial branch of the radial nerve courses along the radialforearm deep to the brachioradialis muscle. The nerve pierces thesuperficial forearm fascia near the junction of the middle anddistal thirds of the forearm between brachioradialis and theextensor carpi radialis longus. It can be entrapped by either themargin of the brachioradialis or extensor carpi radialis longus orin the fascia between these two tendons.

101. A 28-year-old pipe fitter recently recovered from a distalradial fracture in his dominant hand and went back to work. Henow complains of ulnar-sided wrist pain with griping, tool use,and especially with twisting activities. He is having a difficult timestaying on the job. Your examination demonstrates mild swelling,but moderate tenderness over the ulnar styloid and instability ofthe DRUJ. Treatment should consist of:

1) Physical therapy and work hardening 2) Wrist brace 3) Open reduction and internal fixation of the ulnar styloid 4) Excision of the ulnar styloid 5) Excision of the ulnar styloid and TFCC repair

Preferred Response: 5

Discussion:

Because the fragment is small, it must be excised rather thanrepaired. Since the distal radio ulnar joint is unstable andsymptomatic, simple excision is not enough. The TFCCperipheral tear must also be repaired to achieve a successfulresult. A brace may provide some support and relief, but does not

Page 66: ASSH Hand 1998

definitively address the problem. Neither bracing nor intensivetherapy will correct this problem and neither is likely to provideenough relief in the dominant hand of a skilled manual worker toallow him to return to unrestricted work within his pain tolerance.

102. What is the most common cause of compression of theulnar nerve at the level of the wrist or through Guyon's canal?

1) Aneurysm 2) Muscle anomaly 3) Ganglion 4) Fracture 5) Thickened fascia

Preferred Response: 3

Discussion:

Unlike carpal tunnel syndrome where most cases are idiopathic,there is usually an identifiable cause for compression of the ulnarnerve at the wrist or through Guyon's canal. Fractures of thehook of the hamate can injure the ulnar nerve, typically the motorbranch. Anomalous muscles can lead to compression, and directtrauma can lead to an acute chronic nerve injury. Thickened fasciaor an abnormality of the roof of Guyon's canal have not beendescribed. Ganglions in Guyon's canal can cause isolatedsensory or motor nerve deficits or a combination of both.

103. A 52-year-old male with rheumatoid arthritis has mild wristpain, minimal wrist deformity and a functional range of wristmotion. Radiographs show early ulnar translocation of thecarpus. The surgical procedure that will arrest and correct theulnar translocation while preserving maximum wrist function is:

1) Total wrist fusion 2) Darrach procedure 3) Radiolunate fusion 4) Sauve-Kapandji procedure 5) Lunate-capitate-hamate-triquetrum fusion

Preferred Response: 3

Discussion:

A total wrist fusion will eliminate ulnar translocation but will notpreserve wrist function. Distal ulna procedures such as the

Page 67: ASSH Hand 1998

Darrach distal ulna resection and the Sauve-Kapandji distal radioulnar joint reconstruction have not been shown to preventprogressive ulnar translocation. Fusion of the radius to the lunatewill correct ulnar translocation while preserving some wristmotion through the mid-carpal joints. The SauveKapandjiprocedure may arrest ulnar translocation according to someauthors, but will not correct existing ulnar translocation.

104. A 12-year-old girl with spastic hemiplegia due to cerebralpalsy undergoes FCU to ECRB tendon transfer to correct a wristflexion deformity. Postoperatively she has difficulty releasingobjects. This is most likely due to:

1) Co-contraction of digital flexors and extensors. 2) Co-contraction of FCU and ECU causing wrist extensiondeformity. 3) Weak digital extensors. 4) Excessive tension of transferred FCU. 5) Tightness of the digital flexors.

Preferred Response: 3

Discussion:

The Green transfer (FCU to ECRB) is indicated in patients withwrist flexion deformity that have active digital extension with thewrist in neutral position. For those with weak digital extensionthat require the tenodesis effect of wrist flexion for release, thistransfer is contra-indicated since release will be impairedpostoperatively.

105. A 23-year-old auto mechanic sustained a crush injury of hisright hand that resulted in a fracture of the second metacarpal.This was treated by ORIF using a plate and screws. The fractureheals, but the patient complains of limited index finger flexion.On examination, the PIP joint extends when the MP joint isplaced in flexion. PIP flexion is normal when the MCP joint isheld in neutral. This patient's problem is caused by:

1) flexor tendon adherence 2) intrinsic contracture 3) altered extensor moment arm by the plate 4) extensor tendon adherence 5) malunion of the metacarpal

Page 68: ASSH Hand 1998

Preferred Response: 4

Discussion:

The extrinsic extensor tendon passes across both the MP andPIP joints before inserting into the base of the middle phalanx.Thus, it can extend both the MP and PIP joints. A metacarpalfracture can cause adherence of the extensor tension at thefracture site. This tethers the extensor tendon limiting the normaltendon excursion and preventing simultaneous flexion of the MPand PIP joints. Passive MP joint flexion takes up what excursionis available and then tensions the extensor tendon causing the PIPjoint to extend. Conversely, passive PIP joint flexion causes theMP to extend. These maneuvers constitute the clinical tests forextrinsic tendon adherence.

106. Exploration of a sharp transverse laceration to the volarsurface of middle finger just proximal to the PIP jointdemonstrates an isolated 75% laceration of the FDP tendon.Appropriate management of injury includes:

1) Wound closure; immediate mobilization program. 2) Tendon repair; immobilization for three weeks. 3) Tendon repair; immediate controlled mobilization. 4) Completion of the tendon laceration; immediate repair;immobilization for three weeks. 5) Completion of the tendon laceration, immediate repair;immediate controlled mobilization.

Preferred Response: 3

Discussion:

The results of Partial Zone II flexor tendon injuries involving>60% of the cross-sectional area of the tendon are better if theyare surgically repaired. Completion of the tendon lacerationwould disrupt what remains of the existing intrinsic blood supplyof the tendon and is therefore not recommended. Numerousstudies have also demonstrated the advantages of earlymobilization in the treatment of Zone II flexor tendon injuries.

107. The fragment in an intra-articular fracture (Bennett type) ofthe thumb metacarpal is located where?

1) volar and ulnar

Page 69: ASSH Hand 1998

2) central 3) dorsal and ulnar 4) volar and radial 5) dorsal and radial

Preferred Response: 1

Discussion:

The Bennett "fragment" is attached to the anterior (volar) obliqueligament. This ligament is anterior and runs from the ulnar baseof the metacarpal to the trapezium. It is the primary stabilizer ofthe trapeziometacarpal joint.

108. A 50-year-old marine biologist presents with a six-monthhistory of pain and swelling in the proximal interphalangeal jointof his index finger (Figure 21A). Radiographs show soft-tissueswelling about the joint. A biopsy is performed (Figure 21B).Treatment should include:

1) oxacillin and an aminoglycoside 2) allopurinol 3) prednisone 4) aspirin, gold, methotrexate 5) rifampin, INH, ethambutol

Preferred Response: 5

Discussion:

This patient has a granulomatous infection of the proximalinterphalangeal joint which is consistent with an atypicalmycobacterial infection. This man has an occupation that requiresaquatic exposure and M. marinum synovitis should be suspected.

Page 70: ASSH Hand 1998

Radiographs that show soft-tissue swelling but no articulardestruction make a diagnosis of pyogenic infection or anoninfectious inflammatory arthropathy (like rheumatoidarthritis) unlikely. Gout histologically shows amorphous tophuswith surrounding giant cells and variable inflammatory infiltrate.Atypical mycobacterial infections are granulomatous with giantcells and histocytes. Deep atypical mycobacterial infectionsrequire both surgical debridement and appropriate antibiotictherapy for nine to 12 months.

109. A six-year-old child presents for evaluation of her hands.She is somewhat obese and has had several operations on herskull to treat synostosis of the coronal, sagittal, and lamboidsutures. Examination of her hands reveals ulnar sidedpolydactyly, incomplete simple syndactyly of all fingers and thethumb abnormality shown in Figure 22. The most likelydiagnosis is which syndrome?

1) Apert's 2) Lawrence-Moon-Biedl 3) Carpenter 4) Ellis Van Creveld 5) Down's

Preferred Response: 3

Discussion:

The Carpenter syndrome was not firmly established as an entityuntil Temtamy's report in 1966. These patients have often been

Page 71: ASSH Hand 1998

mislabeled in the literature as examples of Apert's syndrome orthe LawrenceMoon-Biedl syndrome. This syndrome is thoughtto be inherited as an autosomal recessive condition. Thecharacteristic features of this syndrome include obesity andduplication of the proximal phalanx of the thumb. This child alsohas "kissing delta" bones. Additional findings commonly includeclinodactly, incomplete syndactyly, camptodactyly, and a singleflexion crease. Polydactyly of the great toes, hypogenitalia, andheart defects may also be seen. Mental retardation is not anobligate feature of the syndrome and does not relate to the timingof craniofacial surgery. Synostosis of the cranial sutures is alsoseen in Apert's syndrome but these children never havepolydactyly. Although all of the other syndromes listed mayinclude polydactyly of the little finger or thumb except Apert's,none of these conditions is characterized by isolated duplicationof the proximal phalanges of the thumb.

110. Rerouting of the extensor pollicis longus tendon through thefirst dorsal compartment may be used as the sole procedure totreat mild spastic thumb-in-palm deformity. Which of thefollowing is a contraindication to the use of EPL reroutingprocedure as the sole corrective procedure.

1) Stable MCP joint 2) Adequate web space 3) Mild EPL weakness 4) Adductor pollicis contracture 5) Abductor pollicis brevis spasticity

Preferred Response: 4

Discussion:

Rerouting the EPL tendon through the first dorsal compartmentalters its vector to coincide with that of the EPB and APLmuscles. A mildly weak EPL is not a contraindication to thisprocedure because this transfer functions partly by a tenodesiseffect. Prerequisites for a satisfactory outcome include fullpassive thumb abduction and no adductor or first dorsalinterosseous spasticity.

111. In hands affected by rheumatoid arthritis, lateral bandrelease (surgical separation of the lateral bands from the centralslip of the extensor tendon) is indicated for the treatment of

Page 72: ASSH Hand 1998

which problem?

1) Boutonniere deformity 2) Swan-neck deformity 3) Extensor tendon rupture 4) Flexor tendon rupture 5) Type II thumb deformity

Preferred Response: 2

Discussion:

In swan-neck deformity without advanced joint destruction, thelateral bands may adhere to the central slip of the extensortendon. These adherences limit volar excursion of the lateralbands during PIP flexion and thereby interfere with PIP jointflexion. Surgical separation of the lateral bands from the centralslip combined with manipulation of the PIP joint into flexion andtemporary pinning can improve PIP joint function in thesefingers.

112. Intramuscular injection of Botulinum toxin may be indicatedin which of the following:

1) 35-year-old female with myasthenia gravis and a forearmpronation contracture. 2) Prior to wrist arthrodesis in a 20-year-old male with brachialplexus palsy and a wrist flexion contracture. 3) Seven-year-old girl with cerebral palsy and a thumb adductioncontracture. 4) 12-year-old boy with Werdnig-Hoffman disease and bilateralelbow flexion contractures. 5) 27-year-old quadriplegic to reduce painful spasm in the wristextensors.

Preferred Response: 3

Discussion:

Botulinum toxin is a protein that reversibly blocks the release ofacetylcholine in muscle and causes paralysis. It has clinicalapplication for the reduction of muscle tone and spasm. It is usedprimarily in orthopaedics in the treatment of spastic conditionsdue to cerebral palsy and traumatic brain injury. In this case thecorrect potential application is in the child with a thumb adductioncontracture due to cerebral palsy. None of the other situations

Page 73: ASSH Hand 1998

would be appropriate for Botulinum toxin .

113. A 35-year-old banker jams his right middle finger whileattempting to catch a softball bare-handed. He seeks no treatment.Two months later he complains that he cannot fully extend theDIP joint of the injured finger. Treatment should consist of:

1) repairing the terminal tendon. 2) full-time DIP joint extension splinting. 3) reconstruction of the terminal tendon. 4) observation. 5) DIP joint fusion.

Preferred Response: 2

Discussion:

Mallet fingers without joint subluxation even in the presence offracture usually do extremely well with eight weeks of full-timeextension splinting followed by gradual mobilization of the joint.Surgery is rarely needed and should be reserved for failure ofconservative treatment. Conservative treatment is frequentlysuccessful as late as three months and on occasion even at sixmonths following injury.

114. Which artery is the most common source of emboli ofarterial origin in the upper extremity?

1) Subclavian. 2) Brachial. 3) Ulnar. 4) Radial. 5) Digital.

Preferred Response: 1

Discussion:

Most emboli originating from an artery start from the subclavianartery. Signs and symptoms of a peripheral embolus are usuallythe sudden onset of pain followed by pallor, coldness,paresthesias, absence of distal pulses, and variable paralysis.

115. In a patient with psoriasis and arthritis mutilans affecting theinterphalangeal joints of the dominant index finger, the

Page 74: ASSH Hand 1998

appropriate treatment is:

1) DIP fusion and PIP joint arthroplasty 2) DIP fusion and PIP joint fusion 3) Steroid injection of DIP and PIP joints 4) Thiotepa injection of DIP and PIP joints 5) Oral methotrexate

Preferred Response: 2

Discussion:

Arthritis mutilans is described as severe progressive loss of bonestock with collapse and shortening of the digits that can result inthe so-called "opera glass" hand. It must be treated with early andaggressive joint fusion, usually with bone graft to restore thenormal length of the digit. There is no role for medical treatmentin arthritis mutilans affecting the interphalangeal joints.

116. A 42-year-old healthy female is seen in clinic one week afterinterceding in a fight between her teenage children. She is unableto flex her ring finger on her dominant left hand. She recalledpain and swelling in the palm of her hand and finger after thefight and noted continued pain at the mid-distal palm overlyingthe fourth ray. There were no breaks in her skin and passivemotion was intact on examination. X-rays revealed no fractures.Her diagnosis is:

1) Avulsion of the insertion of the flexor digitorum profundus tothe ring finger. 2) Avulsion of the flexor digitorum superficialis of the ringfinger. 3) Dislocation of the distal interphalangeal joint of the ringfinger. 4) Hematoma in the flexor tendon sheath, inhibiting motion. 5) Sesamoid dislocation blocking interphalangeal joint motion.

Preferred Response: 1

Discussion:

Loss of distal interphalangeal joint flexion is caused by theavulsion of the flexor digitorum profundus tendon associatedwith painful swelling in the palm from the retracted tendon.Flexor digitorum supefficialis would not affect distal

Page 75: ASSH Hand 1998

interphalangeal joint function and full passive motion is presentobviating dislocation of the distal interphalangeal joint.Hematoma in the palm would not cause loss of flexion andsesamoid dislocation is unlikely and not present in view ofnormal X-rays.

117. Which of the following is not a cause of PIP jointcontracture in Dupuytren's disease?

1) Spiral cord 2) Flexor tendon sheath 3) Retrovascular cord 4) Cleland's ligament 5) Check-rein ligaments

Preferred Response: 4

Discussion:

The spiral cord and retrovascular cord are direct causes of PIPjoint contracture in Dupuytren's. Chronic flexion of the PIP jointresults in secondary contracture of the flexor tendon sheath andcheck-rein ligaments of the volar plate. Cleland's ligament is notinvolved in Dupuytren's disease nor does it produce a secondaryPIP joint contracture. 118. Distal ulnar resection in rheumatoid arthritis has beenshown:

1) To have no effect on ulnar translocation of the carpus. 2) To be associated with significant carpal collapse. 3) To be associated with increased radial rotation of the carpus. 4) To diminish the development of spontaneous radiolunatefusion. 5) To increase the risk of extensor tendon rupture.

Preferred Response: 1

Discussion:

Rheumatoid arthritic patients who underwent excision of thedistal ulna were reviewed and were noted not to have associatedstatistically significant collapse, ulnar translocation or radialrotation of the carpus. Sixty-one percent of the wristsspontaneously developed a radial shelf or limited radiocarpalfusion following excision of the distal ulna. Excision of the distalulna in treatment of caput ulna syndrome would diminish the riskof extensor tendon rupture.

Page 76: ASSH Hand 1998

119. Which of the following anatomic structures is not a majorcontributor to the production of the spiral cord in Dupuytren'scontracture.

1) Pretendinous band 2) Spiral band 3) Natatory ligament 4) Lateral digital sheet 5) Grayson's ligament

Preferred Response: 3

Discussion:

The spiral cord results from the progressive fibromatosisinvolvement of the pretendinous band, spiral band, lateral digitalsheet, and Grayson's ligament. Although the natatory ligamentcontributes some fibers to the lateral digital sheet and maybecome involved in the Dupuytren's fibromatosis producing aweb space contracture, it is not involved structurally in thedevelopment of the spiral cord.

120. A 25-year-old male sustains a ring avulsion injury shown inFigure 23. On examination, the flexor digitorum sublimis is intactas is the central slip of the extensor mechanism of the finger.There are no fractures. Recommended treatment is:

1) revascularization with appropriate vein and/or artery repair. 2) amputation of the ring finger at the MCP joint level. 3) amputation of the ring finger at the PIP joint level. 4) ray amputation of the ring finger with deep transversemetacarpal ligament repair. 5) ray amputation of the ring finger with small to ring metacarpaltransposition.

Preferred Response: 1

Page 77: ASSH Hand 1998

Discussion:

This figure represents a Urbaniak Class II ring-avulsion injury.Such injuries may be reconstructed and satisfactory resultsobtained, particularly when the injury is distal to Zone 2. With aflexor digitorum sublimis and central slip of the extensormechanism intact, satisfactory range of motion of the PIP jointcan be expected. A swan-neck deformity may develop and couldbe reconstructed if necessary. Amputation of the ring finger at theMCP, PIP joint, or entire ray level could be considered, but this isa young, otherwise healthy individual, and therefore,revascularization is a better choice. Ray amputation andmetacarpal transposition is undesirable as it would requirehealing of a metacarpal which is not presently injured